Programs & Examples On #Google font api

The Google Font Directory provides high-quality web fonts that you can include in your pages using the Google Font API. Available at http://code.google.com/apis/webfonts/

How to use font-family lato?

Please put this code in head section

<link href='http://fonts.googleapis.com/css?family=Lato:400,700' rel='stylesheet' type='text/css'>

and use font-family: 'Lato', sans-serif; in your css. For example:

h1 {
    font-family: 'Lato', sans-serif;
    font-weight: 400;
}

Or you can use manually also

Generate .ttf font from fontSquiral

and can try this option

    @font-face {
        font-family: "Lato";
        src: url('698242188-Lato-Bla.eot');
        src: url('698242188-Lato-Bla.eot?#iefix') format('embedded-opentype'),
        url('698242188-Lato-Bla.svg#Lato Black') format('svg'),
        url('698242188-Lato-Bla.woff') format('woff'),
        url('698242188-Lato-Bla.ttf') format('truetype');
        font-weight: normal;
        font-style: normal;
}

Called like this

body {
  font-family: 'Lato', sans-serif;
}

How to use Google fonts in React.js?

Google fonts in React.js?

Open your stylesheet i.e, app.css, style.css (what name you have), it doesn't matter, just open stylesheet and paste this code

@import url('https://fonts.googleapis.com/css?family=Josefin+Sans');

and don't forget to change URL of your font that you want, else working fine

and use this as :

body {
  font-family: 'Josefin Sans', cursive;
}

How to host google web fonts on my own server?

There is a very simple script, written in plain Java, to download all fonts from a Google Web Font link (multiple fonts supported). It also downloads the CSS file and adapts it to local files. The user-agent can be adapted to get also other files than only WOFF2. See https://github.com/ssc-hrep3/google-font-download

The resulting files can easily be added to a build process (e.g. a webpack build like vue-webpack).

Received fatal alert: handshake_failure through SSLHandshakeException

This issue is occurring because of the java version. I was using 1.8.0.231 JDK and getting this error. I have degraded my java version from 1.8.0.231 to 1.8.0.171, Now It is working fine.

jQuery append() - return appended elements

I think you could do something like this:

var $child = $("#parentId").append("<div></div>").children("div:last-child");

The parent #parentId is returned from the append, so add a jquery children query to it to get the last div child inserted.

$child is then the jquery wrapped child div that was added.

How to show the last queries executed on MySQL?

Maybe you could find that out by looking at the query log.

Java Ordered Map

tl;dr

To keep Map< Integer , String > in an order sorted by key, use either of the two classes implementing the SortedMap/NavigableMap interfaces:

  • TreeMap
  • ConcurrentSkipListMap

If manipulating the map within a single thread, use the first, TreeMap. If manipulating across threads, use the second, ConcurrentSkipListMap.

For details, see the table below and the following discussion.

Details

Here is a graphic table I made showing the features of the ten Map implementations bundled with Java 11.

The NavigableMap interface is what SortedMap should have been in the first place. The SortedMap logically should be removed but cannot be as some 3rd-party map implementations may be using interface.

As you can see in this table, only two classes implement the SortedMap/NavigableMap interfaces:

Both of these keep keys in sorted order, either by their natural order (using compareTo method of the Comparable(https://docs.oracle.com/en/java/javase/11/docs/api/java.base/java/lang/Comparable.html) interface) or by a Comparator implementation you pass. The difference between these two classes is that the second one, ConcurrentSkipListMap, is thread-safe, highly concurrent.

See also the Iteration order column in the table below.

  • The LinkedHashMap class returns its entries by the order in which they were originally inserted.
  • EnumMap returns entries in the order by which the enum class of the key is defined. For example, a map of which employee is covering which day of the week (Map< DayOfWeek , Person >) uses the DayOfWeek enum class built into Java. That enum is defined with Monday first and Sunday last. So entries in an iterator will appear in that order.

The other six implementations make no promise about the order in which they report their entries.

Table of map implementations in Java 11, comparing their features

Comparing date part only without comparing time in JavaScript

I know this question have been already answered and this may not be the best way, but in my scenario its working perfectly, so I thought it may help someone like me.

if you have date string as

String dateString="2018-01-01T18:19:12.543";

and you just want to compare the date part with another Date object in JS,

var anotherDate=new Date(); //some date

then you have to convert the string to Date object by using new Date("2018-01-01T18:19:12.543");

and here is the trick :-

var valueDate =new Date(new Date(dateString).toDateString());

            return valueDate.valueOf() == anotherDate.valueOf(); //here is the final result

I have used toDateString() of Date object of JS, which returns the Date string only.

Note: Don't forget to use the .valueOf() function while comparing the dates.

more info about .valeOf() is here reference

Happy codding.

How to drop unique in MySQL?

ALTER TABLE 0_value_addition_setup  DROP  INDEX   value_code

Gson: Directly convert String to JsonObject (no POJO)

Try to use getAsJsonObject() instead of a straight cast used in the accepted answer:

JsonObject o = new JsonParser().parse("{\"a\": \"A\"}").getAsJsonObject();

Add animated Gif image in Iphone UIImageView

This doesn't meet the requirement of using a UIImageView, but maybe this would simplify things for you. Have you considered using a UIWebView?

NSString *gifUrl = @"http://gifs.com";
NSURL *url = [NSURL URLWithString: gifUrl];
[webView loadRequest: [NSURLRequest requestWithURL:url]

If you want, instead of linking to a URL that requires Internet, you could import an HTML file into your Xcode project and set the root in the string.

Xcode doesn't see my iOS device but iTunes does

Just unplug the cable of iPhone with your mac and then plug cable in mac work for me.I hope it's work for someone.

Provide schema while reading csv file as a dataframe

here my solution is:

import org.apache.spark.sql.types._
  val spark = org.apache.spark.sql.SparkSession.builder.
  master("local[*]").
  appName("Spark CSV Reader").
  getOrCreate()

val movie_rating_schema = StructType(Array(
  StructField("UserID", IntegerType, true),
  StructField("MovieID", IntegerType, true),
  StructField("Rating", DoubleType, true),
  StructField("Timestamp", TimestampType, true)))

val df_ratings: DataFrame = spark.read.format("csv").
  option("header", "true").
  option("mode", "DROPMALFORMED").
  option("delimiter", ",").
  //option("inferSchema", "true").
  option("nullValue", "null").
  schema(movie_rating_schema).
  load(args(0)) //"file:///home/hadoop/spark-workspace/data/ml-20m/ratings.csv"

val movie_avg_scores = df_ratings.rdd.map(_.toString()).
  map(line => {
    // drop "[", "]" and then split the str 
    val fileds = line.substring(1, line.length() - 1).split(",")
    //extract (movie id, average rating)
    (fileds(1).toInt, fileds(2).toDouble)
  }).
  groupByKey().
  map(data => {
    val avg: Double = data._2.sum / data._2.size
    (data._1, avg)
  })

call a static method inside a class?

Let's assume this is your class:

class Test
{
    private $baz = 1;

    public function foo() { ... }

    public function bar() 
    {
        printf("baz = %d\n", $this->baz);
    }

    public static function staticMethod() { echo "static method\n"; }
}

From within the foo() method, let's look at the different options:

$this->staticMethod();

So that calls staticMethod() as an instance method, right? It does not. This is because the method is declared as public static the interpreter will call it as a static method, so it will work as expected. It could be argued that doing so makes it less obvious from the code that a static method call is taking place.

$this::staticMethod();

Since PHP 5.3 you can use $var::method() to mean <class-of-$var>::; this is quite convenient, though the above use-case is still quite unconventional. So that brings us to the most common way of calling a static method:

self::staticMethod();

Now, before you start thinking that the :: is the static call operator, let me give you another example:

self::bar();

This will print baz = 1, which means that $this->bar() and self::bar() do exactly the same thing; that's because :: is just a scope resolution operator. It's there to make parent::, self:: and static:: work and give you access to static variables; how a method is called depends on its signature and how the caller was called.

To see all of this in action, see this 3v4l.org output.

What is the Gradle artifact dependency graph command?

If you want recursive to include subprojects, you can always write it yourself:

Paste into the top-level build.gradle:

task allDeps << {
    println "All Dependencies:"
    allprojects.each { p ->
        println()
        println " $p.name ".center( 60, '*' )
        println()
        p.configurations.all.findAll { !it.allDependencies.empty }.each { c ->
            println " ${c.name} ".center( 60, '-' )
            c.allDependencies.each { dep ->
                println "$dep.group:$dep.name:$dep.version"
            }
            println "-" * 60
        }
    }
}

Run with:

gradle allDeps

Android-java- How to sort a list of objects by a certain value within the object

Follow this code to sort any ArrayList

Collections.sort(myList, new Comparator<EmployeeClass>(){
    public int compare(EmployeeClass obj1, EmployeeClass obj2) {
        // ## Ascending order
        return obj1.firstName.compareToIgnoreCase(obj2.firstName); // To compare string values
        // return Integer.valueOf(obj1.empId).compareTo(Integer.valueOf(obj2.empId)); // To compare integer values

        // ## Descending order
        // return obj2.firstName.compareToIgnoreCase(obj1.firstName); // To compare string values
        // return Integer.valueOf(obj2.empId).compareTo(Integer.valueOf(obj1.empId)); // To compare integer values
        }
    });

How to always show the vertical scrollbar in a browser?

I have been doing it this way:

.element {
    overflow-y: visible;
}

Painfully simple I know...

Ansible: filter a list by its attributes

Not necessarily better, but since it's nice to have options here's how to do it using Jinja statements:

- debug:
    msg: "{% for address in network.addresses.private_man %}\
        {% if address.type == 'fixed' %}\
          {{ address.addr }}\
        {% endif %}\
      {% endfor %}"

Or if you prefer to put it all on one line:

- debug:
    msg: "{% for address in network.addresses.private_man if address.type == 'fixed' %}{{ address.addr }}{% endfor %}"

Which returns:

ok: [localhost] => {
    "msg": "172.16.1.100"
}

How can I populate a select dropdown list from a JSON feed with AngularJS?

The proper way to do it is using the ng-options directive. The HTML would look like this.

<select ng-model="selectedTestAccount" 
        ng-options="item.Id as item.Name for item in testAccounts">
    <option value="">Select Account</option>
</select>

JavaScript:

angular.module('test', []).controller('DemoCtrl', function ($scope, $http) {
    $scope.selectedTestAccount = null;
    $scope.testAccounts = [];

    $http({
            method: 'GET',
            url: '/Admin/GetTestAccounts',
            data: { applicationId: 3 }
        }).success(function (result) {
        $scope.testAccounts = result;
    });
});

You'll also need to ensure angular is run on your html and that your module is loaded.

<html ng-app="test">
    <body ng-controller="DemoCtrl">
    ....
    </body>
</html>

How can I generate an HTML report for Junit results?

There are multiple options available for generating HTML reports for Selenium WebDriver scripts.

1. Use the JUNIT TestWatcher class for creating your own Selenium HTML reports

The TestWatcher JUNIT class allows overriding the failed() and succeeded() JUNIT methods that are called automatically when JUNIT tests fail or pass.

The TestWatcher JUNIT class allows overriding the following methods:

  • protected void failed(Throwable e, Description description)

failed() method is invoked when a test fails

  • protected void finished(Description description)

finished() method is invoked when a test method finishes (whether passing or failing)

  • protected void skipped(AssumptionViolatedException e, Description description)

skipped() method is invoked when a test is skipped due to a failed assumption.

  • protected void starting(Description description)

starting() method is invoked when a test is about to start

  • protected void succeeded(Description description)

succeeded() method is invoked when a test succeeds

See below sample code for this case:

import static org.junit.Assert.assertTrue;
import org.junit.Test;

public class TestClass2 extends WatchManClassConsole {

    @Test public void testScript1() {
        assertTrue(1 < 2); >
    }

    @Test public void testScript2() {
        assertTrue(1 > 2);
    }

    @Test public void testScript3() {
        assertTrue(1 < 2);
    }

    @Test public void testScript4() {
        assertTrue(1 > 2);
    }
}

import org.junit.Rule; 
import org.junit.rules.TestRule; 
import org.junit.rules.TestWatcher; 
import org.junit.runner.Description; 
import org.junit.runners.model.Statement; 

public class WatchManClassConsole {

    @Rule public TestRule watchman = new TestWatcher() { 

        @Override public Statement apply(Statement base, Description description) { 
            return super.apply(base, description); 
        } 

        @Override protected void succeeded(Description description) { 
            System.out.println(description.getDisplayName() + " " + "success!"); 
        } 

        @Override protected void failed(Throwable e, Description description) { 
            System.out.println(description.getDisplayName() + " " + e.getClass().getSimpleName()); 
        }
    }; 
}

2. Use the Allure Reporting framework

Allure framework can help with generating HTML reports for your Selenium WebDriver projects.

The reporting framework is very flexible and it works with many programming languages and unit testing frameworks.

You can read everything about it at http://allure.qatools.ru/.

You will need the following dependencies and plugins to be added to your pom.xml file

  1. maven surefire
  2. aspectjweaver
  3. allure adapter

See more details including code samples on this article: http://test-able.blogspot.com/2015/10/create-selenium-html-reports-with-allure-framework.html

How to print out all the elements of a List in Java?

public static void main(String[] args) {
        answer(10,60);

    }
    public static void answer(int m,int k){
        AtomicInteger n = new AtomicInteger(m);
        Stream<Integer> stream = Stream.generate(() -> n.incrementAndGet()).limit(k);
        System.out.println(Arrays.toString(stream.toArray()));
    }

c# datatable insert column at position 0

You can use the following code to add column to Datatable at postion 0:

    DataColumn Col   = datatable.Columns.Add("Column Name", System.Type.GetType("System.Boolean"));
    Col.SetOrdinal(0);// to put the column in position 0;

Using multiprocessing.Process with a maximum number of simultaneous processes

more generally, this could also look like this:

import multiprocessing
def chunks(l, n):
    for i in range(0, len(l), n):
        yield l[i:i + n]

numberOfThreads = 4


if __name__ == '__main__':
    jobs = []
    for i, param in enumerate(params):
        p = multiprocessing.Process(target=f, args=(i,param))
        jobs.append(p)
    for i in chunks(jobs,numberOfThreads):
        for j in i:
            j.start()
        for j in i:
            j.join()

Of course, that way is quite cruel (since it waits for every process in a junk until it continues with the next chunk). Still it works well for approx equal run times of the function calls.

Solution to INSTALL_FAILED_INSUFFICIENT_STORAGE error on Android

Just uninstall the application from emulator either from command line or go to settings and uninstall the application. This will stop the error from coming.

How do I ALTER a PostgreSQL table and make a column unique?

Or, have the DB automatically assign a constraint name using:

ALTER TABLE foo ADD UNIQUE (thecolumn);

Difference between CLOB and BLOB from DB2 and Oracle Perspective?

They can be considered as equivalent. The limits in size are the same:

  • Maximum length of CLOB (in bytes or OCTETS)) 2 147 483 647
  • Maximum length of BLOB (in bytes) 2 147 483 647

There is also the DBCLOBs, for double byte characters.

References:

iTunes Connect Screenshots Sizes for all iOS (iPhone/iPad/Apple Watch) devices

Now Apple Inc. added a new device screen shots also over iTunesconnect that is iPad Pro. Here are all sizes of screen shots which iTunesconnects requires.

  • iPhone 6 Plus (5.5 inches) - 2208x1242
  • iPhone 6 (4.7 inches) - 1334x750
  • iPhone 5/5s (4 inches) - 1136x640
  • iPhone 4s (3.5 inches) - 960x640
  • iPad - 1024x768
  • iPadPro - 2732x2048

How do I set the timeout for a JAX-WS webservice client?

the easiest way to avoid slow retrieval of the remote WSDL when you instantiate your SEI is to not retrieve the WSDL from the remote service endpoint at runtime.

this means that you have to update your local WSDL copy any time the service provider makes an impacting change, but it also means that you have to update your local copy any time the service provider makes an impacting change.

When I generate my client stubs, I tell the JAX-WS runtime to annotate the SEI in such a way that it will read the WSDL from a pre-determined location on the classpath. by default the location is relative to the package location of the Service SEI


<wsimport
    sourcedestdir="${dao.helter.dir}/build/generated"
    destdir="${dao.helter.dir}/build/bin/generated"
    wsdl="${dao.helter.dir}/src/resources/schema/helter/helterHttpServices.wsdl"
    wsdlLocation="./wsdl/helterHttpServices.wsdl"
    package="com.helter.esp.dao.helter.jaxws"
    >
    <binding dir="${dao.helter.dir}/src/resources/schema/helter" includes="*.xsd"/>
</wsimport>
<copy todir="${dao.helter.dir}/build/bin/generated/com/helter/esp/dao/helter/jaxws/wsdl">
    <fileset dir="${dao.helter.dir}/src/resources/schema/helter" includes="*" />
</copy>

the wsldLocation attribute tells the SEI where is can find the WSDL, and the copy makes sure that the wsdl (and supporting xsd.. etc..) is in the correct location.

since the location is relative to the SEI's package location, we create a new sub-package (directory) called wsdl, and copy all the wsdl artifacts there.

all you have to do at this point is make sure you include all *.wsdl, *.xsd in addition to all *.class when you create your client-stub artifact jar file.

(in case your curious, the @webserviceClient annotation is where this wsdl location is actually set in the java code

@WebServiceClient(name = "httpServices", targetNamespace = "http://www.helter.com/schema/helter/httpServices", wsdlLocation = "./wsdl/helterHttpServices.wsdl")

Get combobox value in Java swing

If the string is empty, comboBox.getSelectedItem().toString() will give a NullPointerException. So better to typecast by (String).

How do I find the length/number of items present for an array?

Do you mean how long is the array itself, or how many customerids are in it?

Because the answer to the first question is easy: 5 (or if you don't want to hard-code it, Ben Stott's answer).

But the answer to the other question cannot be automatically determined. Presumably you have allocated an array of length 5, but will initially have 0 customer IDs in there, and will put them in one at a time, and your question is, "how many customer IDs have I put into the array?"

C can't tell you this. You will need to keep a separate variable, int numCustIds (for example). Every time you put a customer ID into the array, increment that variable. Then you can tell how many you have put in.

How to print an exception in Python 3?

[In Python3]

Let's say you want to handle an IndexError and print the traceback, you can do the following:

from traceback import print_tb 
empty_list = [] 
try: 
    x = empty_list[100]
except IndexError as index_error: 
    print_tb(index_error.__traceback__)

Note: You can use the format_tb function instead of print_tb to get the traceback as a string for logging purposes. Hope this helps.

How to force addition instead of concatenation in javascript

Should also be able to do this:

total += eval(myInt1) + eval(myInt2) + eval(myInt3);

This helped me in a different, but similar, situation.

HTTP Basic Authentication credentials passed in URL and encryption

Will the username and password be automatically SSL encrypted? Is the same true for GETs and POSTs

Yes, yes yes.

The entire communication (save for the DNS lookup if the IP for the hostname isn't already cached) is encrypted when SSL is in use.

inline conditionals in angular.js

Angular 1.1.5 added support for ternary operators:

{{myVar === "two" ? "it's true" : "it's false"}}

Using multiple case statements in select query

There are two ways to write case statements, you seem to be using a combination of the two

case a.updatedDate
    when 1760 then 'Entered on' + a.updatedDate
    when 1710 then 'Viewed on' + a.updatedDate
    else 'Last Updated on' + a.updateDate
end

or

case 
    when a.updatedDate = 1760 then 'Entered on' + a.updatedDate
    when a.updatedDate = 1710 then 'Viewed on' + a.updatedDate
    else 'Last Updated on' + a.updateDate
end

are equivalent. They may not work because you may need to convert date types to varchars to append them to other varchars.

PermissionError: [WinError 5] Access is denied python using moviepy to write gif

I've run into this as well, solution is usually to be sure to run the program as an administrator (right click, run as administrator.)

SQL Server: Query fast, but slow from procedure

-- Here is the solution:

create procedure GetOrderForCustomers(@CustID varchar(20))

as

begin

select * from orders

where customerid = ISNULL(@CustID, '')

end

-- That's it

Debian 8 (Live-CD) what is the standard login and password?

I am using Debian 8 live off a USB. I was locked out of the system after 10 min of inactivity. The password that was required to log back in to the system for the user was:

login : Debian Live User
password : live

I hope this helps

Mockito: List Matchers with generics

In addition to anyListOf above, you can always specify generics explicitly using this syntax:

when(mock.process(Matchers.<List<Bar>>any(List.class)));

Java 8 newly allows type inference based on parameters, so if you're using Java 8, this may work as well:

when(mock.process(Matchers.any()));

Remember that neither any() nor anyList() will apply any checks, including type or null checks. In Mockito 2.x, any(Foo.class) was changed to mean "any instanceof Foo", but any() still means "any value including null".

NOTE: The above has switched to ArgumentMatchers in newer versions of Mockito, to avoid a name collision with org.hamcrest.Matchers. Older versions of Mockito will need to keep using org.mockito.Matchers as above.

How can I submit a POST form using the <a href="..."> tag?

I use a jQuery script to create "shadow" forms for my POSTable links.

Instead of <a href="/some/action?foo=bar">, I write <a data-post="/some/action" data-var-foo="bar" href="#do_action_foo_bar">. The script makes a hidden form with hidden inputs, and submits it when the link is clicked.

_x000D_
_x000D_
$("a[data-post]")
    .each(function() {
        let href = $(this).data("post"); if (!href) return;
        let $form = $("<form></form>").attr({ method:"POST",action:href }).css("display","none")
        let data = $(this).data()
        for (let dat in data) {
            if (dat.startsWith("postVar")) {
                let varname = dat.substring(7).toLowerCase()  // postVarId -> id
                let varval = data[dat]
                $form.append($("<input/>").attr({ type:"hidden",name:varname,value:varval }))
            }
        }
        $("body").append($form)
        $(this).data("postform",$form)
    })
    .click(function(ev) {
        ev.preventDefault()
        if ($(this).data("postform")) $(this).data("postform").submit(); else console.error("No .postform set in <a data-post>")
    })
_x000D_
<script src="https://cdnjs.cloudflare.com/ajax/libs/jquery/3.3.1/jquery.min.js"></script>
<a data-post="/some/action" data-var-foo="bar" href="#do_action_foo_bar">click me</a>
_x000D_
_x000D_
_x000D_

Resetting Select2 value in dropdown with reset button

You can also reset select2 value using

$(function() {
  $('#d').select2('data', null)
})

alternately you can pass 'allowClear': true when calling select2 and it will have an X button to reset its value.

You must enable the openssl extension to download files via https

Becareful if you are using wamp don't use the wamp ui to enable the extension=php_openssl.dll

just go to your php directory , for example : C:\wamp\bin\php\php5.4.12 and edit the php.ini and uncomment the extension=php_openssl.dll.

it should work.

How to git clone a specific tag

Cloning a specific tag, might return 'detached HEAD' state.

As a workaround, try to clone the repo first, and then checkout a specific tag. For example:

repo_url=https://github.com/owner/project.git
repo_dir=$(basename $repo_url .git)
repo_tag=0.5

git clone --single-branch $repo_url # using --depth 1 can show no tags
git --work-tree=$repo_dir --git-dir=$repo_dir/.git checkout tags/$repo_tag

Note: Since Git 1.8.5, you can use -C <path>, instead of --work-tree and --git-dir.

How to determine the Boost version on a system?

cat /usr/local/include/boost/version.hpp | grep BOOST_LIB_VERSION

How do I find out what License has been applied to my SQL Server installation?

When I run:

   exec sp_readerrorlog @p1 = 0
   ,@p2 = 1
   ,@p3 = N'licensing'

I get:

SQL Server detected 2 sockets with 21 cores per socket and 21 logical processors per socket, 42 total logical processors; using 20 logical processors based on SQL Server licensing. This is an informational message; no user action is required.

also, SELECT @@VERSION shows:

Microsoft SQL Server 2014 (SP1-GDR) (KB4019091) - 12.0.4237.0 (X64) Jul 5 2017 22:03:42 Copyright (c) Microsoft Corporation Enterprise Edition (64-bit) on Windows NT 6.3 (Build 9600: ) (Hypervisor)

This is a VM

C# nullable string error

For nullable, use ? with all of the C# primitives, except for string.

The following page gives a list of the C# primitives: http://msdn.microsoft.com/en-us/library/aa711900(v=vs.71).aspx

Silent installation of a MSI package

The proper way to install an MSI silently is via the msiexec.exe command line as follows:

msiexec.exe /i c:\setup.msi /QN /L*V "C:\Temp\msilog.log"

Quick explanation:

 /L*V "C:\Temp\msilog.log"= verbose logging
 /QN = run completely silently
 /i = run install sequence 

There is a much more comprehensive answer here: Batch script to install MSI. This answer provides details on the msiexec.exe command line options and a description of how to find the "public properties" that you can set on the command line at install time. These properties are generally different for each MSI.

How do I connect to mongodb with node.js (and authenticate)?

Everyone should use this source link:

http://mongodb.github.com/node-mongodb-native/contents.html

Answer to the question:

var Db = require('mongodb').Db,
    MongoClient = require('mongodb').MongoClient,
    Server = require('mongodb').Server,
    ReplSetServers = require('mongodb').ReplSetServers,
    ObjectID = require('mongodb').ObjectID,
    Binary = require('mongodb').Binary,
    GridStore = require('mongodb').GridStore,
    Code = require('mongodb').Code,
    BSON = require('mongodb').pure().BSON,
    assert = require('assert');

var db = new Db('integration_tests', new Server("127.0.0.1", 27017,
 {auto_reconnect: false, poolSize: 4}), {w:0, native_parser: false});

// Establish connection to db
db.open(function(err, db) {
  assert.equal(null, err);

  // Add a user to the database
  db.addUser('user', 'name', function(err, result) {
    assert.equal(null, err);

    // Authenticate
    db.authenticate('user', 'name', function(err, result) {
      assert.equal(true, result);

      db.close();
    });
  });
});

How to get a Char from an ASCII Character Code in c#

You can simply write:

char c = (char) 2;

or

char c = Convert.ToChar(2);

or more complex option for ASCII encoding only

char[] characters = System.Text.Encoding.ASCII.GetChars(new byte[]{2});
char c = characters[0];

How can I remove space (margin) above HTML header?

Try:

h1 {
    margin-top: 0;
}

You're seeing the effects of margin collapsing.

Check mySQL version on Mac 10.8.5

Or just call mysql command with --version option.

mysql --version

How to verify if nginx is running or not?

For Mac users

I found out one more way: You can check if /usr/local/var/run/nginx.pid exists. If it is - nginx is running. Useful way for scripting.

Example:

if [ -f /usr/local/var/run/nginx.pid ]; then
   echo "Nginx is running"

fi

Convert from DateTime to INT

EDIT: Casting to a float/int no longer works in recent versions of SQL Server. Use the following instead:

select datediff(day, '1899-12-30T00:00:00', my_date_field)
from mytable

Note the string date should be in an unambiguous date format so that it isn't affected by your server's regional settings.


In older versions of SQL Server, you can convert from a DateTime to an Integer by casting to a float, then to an int:

select cast(cast(my_date_field as float) as int)
from mytable

(NB: You can't cast straight to an int, as MSSQL rounds the value up if you're past mid day!)

If there's an offset in your data, you can obviously add or subtract this from the result

You can convert in the other direction, by casting straight back:

select cast(my_integer_date as datetime)
from mytable

What is the purpose of meshgrid in Python / NumPy?

meshgrid helps in creating a rectangular grid from two 1-D arrays of all pairs of points from the two arrays.

x = np.array([0, 1, 2, 3, 4])
y = np.array([0, 1, 2, 3, 4])

Now, if you have defined a function f(x,y) and you wanna apply this function to all the possible combination of points from the arrays 'x' and 'y', then you can do this:

f(*np.meshgrid(x, y))

Say, if your function just produces the product of two elements, then this is how a cartesian product can be achieved, efficiently for large arrays.

Referred from here

Stripping everything but alphanumeric chars from a string in Python

How about:

def ExtractAlphanumeric(InputString):
    from string import ascii_letters, digits
    return "".join([ch for ch in InputString if ch in (ascii_letters + digits)])

This works by using list comprehension to produce a list of the characters in InputString if they are present in the combined ascii_letters and digits strings. It then joins the list together into a string.

iOS how to set app icon and launch images

In the left list, right click on "AppIcon" and click on "Open in finder" A folder with name "AppIcon.appiconset" will open. Paste all the graphics with required resolution there. Once done, all those images will be visible in this same screen(one in your screen shot). then drag them to appropriate box. App icons have been added. Same process for Launch images. Launch images through this process are added for iOS 7 and below. For iOS 8 separate LaunchScreen.xib file is made by default.

Can constructors be async?

I'm not familiar with the async keyword (is this specific to Silverlight or a new feature in the beta version of Visual Studio?), but I think I can give you an idea of why you can't do this.

If I do:

var o = new MyObject();
MessageBox(o.SomeProperty.ToString());

o may not be done initializing before the next line of code runs. An instantiation of your object cannot be assigned until your constructor is completed, and making the constructor asynchronous wouldn't change that so what would be the point? However, you could call an asynchronous method from your constructor and then your constructor could complete and you would get your instantiation while the async method is still doing whatever it needs to do to setup your object.

Setting the default page for ASP.NET (Visual Studio) server configuration

The built-in webserver is hardwired to use Default.aspx as the default page.

The project must have atleast an empty Default.aspx file to overcome the Directory Listing problem for Global.asax.

:)

Once you add that empty file all requests can be handled in one location.

public class Global : System.Web.HttpApplication
{
    protected void Application_BeginRequest(object sender, EventArgs e)
    {
        this.Response.Write("hi@ " + this.Request.Path + "?" + this.Request.QueryString);
        this.Response.StatusCode = 200;
        this.Response.ContentType = "text/plain";

        this.Response.End();
    }
}

Passing data into "router-outlet" child components

There are 3 ways to pass data from Parent to Children

  1. Through shareable service : you should store into a service the data you would like to share with the children
  2. Through Children Router Resolver if you have to receive different data

    this.data = this.route.snaphsot.data['dataFromResolver'];
    
  3. Through Parent Router Resolver if your have to receive the same data from parent

    this.data = this.route.parent.snaphsot.data['dataFromResolver'];
    

Note1: You can read about resolver here. There is also an example of resolver and how to register the resolver into the module and then retrieve data from resolver into the component. The resolver registration is the same on the parent and child.

Note2: You can read about ActivatedRoute here to be able to get data from router

AngularJS Error: $injector:unpr Unknown Provider

@ user2310334 I just tried this, a VERY basic example:

HTML file

<!DOCTYPE html>
<html lang="en" ng-app="app">

<head>
    <script src="https://cdnjs.cloudflare.com/ajax/libs/angular.js/1.4.6/angular.min.js" type="text/javascript"></script>
    <script src="https://cdnjs.cloudflare.com/ajax/libs/angular.js/1.4.6/angular-route.min.js" type="text/javascript"></script>
    <script src="./app.js" type="text/javascript"></script>
</head>

<body>
    <div ng-controller="MailDetailCtrl">
    </div>
</body>
</html>

The javascript file:

var myApp= angular.module('app', ['ngRoute']);

myApp.factory('mailService' , function () {
return {
    getData : function(){
      var employees = [{name: 'John Doe', id: '1'},
        {name: 'Mary Homes', id: '2'},
        {name: 'Chris Karl', id: '3'}
      ];

      return employees;
    }
  };

});


myApp.controller('MailDetailCtrl',['$scope', 'mailService', function($scope, mailService) {
  alert(mailService.getData()[0].name);
}]);

And it works. Try it.

ImportError: No module named request

You can do that using Python 2.

  1. Remove request
  2. Make that line: from urllib2 import urlopen

You cannot have request in Python 2, you need to have Python 3 or above.

Convert Dictionary to JSON in Swift

My answer for your question is below

let dict = ["0": "ArrayObjectOne", "1": "ArrayObjecttwo", "2": "ArrayObjectThree"]

var error : NSError?

let jsonData = try! NSJSONSerialization.dataWithJSONObject(dict, options: NSJSONWritingOptions.PrettyPrinted)

let jsonString = NSString(data: jsonData, encoding: String.Encoding.utf8.rawValue)! as String

print(jsonString)

Answer is

{
  "0" : "ArrayObjectOne",
  "1" : "ArrayObjecttwo",
  "2" : "ArrayObjectThree"
}

What is the C# equivalent of NaN or IsNumeric?

In addition to the previous correct answers it is probably worth pointing out that "Not a Number" (NaN) in its general usage is not equivalent to a string that cannot be evaluated as a numeric value. NaN is usually understood as a numeric value used to represent the result of an "impossible" calculation - where the result is undefined. In this respect I would say the Javascript usage is slightly misleading. In C# NaN is defined as a property of the single and double numeric types and is used to refer explicitly to the result of diving zero by zero. Other languages use it to represent different "impossible" values.

How to recover deleted rows from SQL server table?

I think thats impossible, sorry.

Thats why whenever running a delete or update you should always use BEGIN TRANSACTION, then COMMIT if successful or ROLLBACK if not.

a tag as a submit button?

in my opinion the easiest way would be somthing like this:

<?php>
echo '<a href="link.php?submit='.$value.'">Submit</a>';
</?>

within the "link.php" you can request the value like this:

$_REQUEST['submit']

ASP.NET MVC - Find Absolute Path to the App_Data folder from Controller

ASP.NET MVC1 -> MVC3

string path = HttpContext.Current.Server.MapPath("~/App_Data/somedata.xml");

ASP.NET MVC4

string path = Server.MapPath("~/App_Data/somedata.xml");


MSDN Reference:

HttpServerUtility.MapPath Method

Array.Add vs +=

If you want a dynamically sized array, then you should make a list. Not only will you get the .Add() functionality, but as @frode-f explains, dynamic arrays are more memory efficient and a better practice anyway.

And it's so easy to use.

Instead of your array declaration, try this:

$outItems = New-Object System.Collections.Generic.List[System.Object]

Adding items is simple.

$outItems.Add(1)
$outItems.Add("hi")

And if you really want an array when you're done, there's a function for that too.

$outItems.ToArray()

How can I convert a string with dot and comma into a float in Python

Just replace, with replace().

f = float("123,456.908".replace(',','')) print(type(f)

type() will show you that it has converted into a float

Can a website detect when you are using Selenium with chromedriver?

Even if you are sending all the right data (e.g. Selenium doesn't show up as an extension, you have a reasonable resolution/bit-depth, &c), there are a number of services and tools which profile visitor behaviour to determine whether the actor is a user or an automated system.

For example, visiting a site then immediately going to perform some action by moving the mouse directly to the relevant button, in less than a second, is something no user would actually do.

It might also be useful as a debugging tool to use a site such as https://panopticlick.eff.org/ to check how unique your browser is; it'll also help you verify whether there are any specific parameters that indicate you're running in Selenium.

Why is it faster to check if dictionary contains the key, rather than catch the exception in case it doesn't?

On the one hand, throwing exceptions is inherently expensive, because the stack has to be unwound etc.
On the other hand, accessing a value in a dictionary by its key is cheap, because it's a fast, O(1) operation.

BTW: The correct way to do this is to use TryGetValue

obj item;
if(!dict.TryGetValue(name, out item))
    return null;
return item;

This accesses the dictionary only once instead of twice.
If you really want to just return null if the key doesn't exist, the above code can be simplified further:

obj item;
dict.TryGetValue(name, out item);
return item;

This works, because TryGetValue sets item to null if no key with name exists.

How to create an array of object literals in a loop?

You can do something like that in ES6.

new Array(10).fill().map((e,i) => {
   return {idx: i}
});

How to resolve cURL Error (7): couldn't connect to host?

In my case, the problem was caused by the hosting provider I was using blocking http packets addressed to their IP block that originated from within their IP block. Un-frickin-believable!!!

How to add noise (Gaussian/salt and pepper etc) to image in Python with OpenCV

The Function adds gaussian , salt-pepper , poisson and speckle noise in an image

Parameters
----------
image : ndarray
    Input image data. Will be converted to float.
mode : str
    One of the following strings, selecting the type of noise to add:

    'gauss'     Gaussian-distributed additive noise.
    'poisson'   Poisson-distributed noise generated from the data.
    's&p'       Replaces random pixels with 0 or 1.
    'speckle'   Multiplicative noise using out = image + n*image,where
                n is uniform noise with specified mean & variance.


import numpy as np
import os
import cv2
def noisy(noise_typ,image):
   if noise_typ == "gauss":
      row,col,ch= image.shape
      mean = 0
      var = 0.1
      sigma = var**0.5
      gauss = np.random.normal(mean,sigma,(row,col,ch))
      gauss = gauss.reshape(row,col,ch)
      noisy = image + gauss
      return noisy
   elif noise_typ == "s&p":
      row,col,ch = image.shape
      s_vs_p = 0.5
      amount = 0.004
      out = np.copy(image)
      # Salt mode
      num_salt = np.ceil(amount * image.size * s_vs_p)
      coords = [np.random.randint(0, i - 1, int(num_salt))
              for i in image.shape]
      out[coords] = 1

      # Pepper mode
      num_pepper = np.ceil(amount* image.size * (1. - s_vs_p))
      coords = [np.random.randint(0, i - 1, int(num_pepper))
              for i in image.shape]
      out[coords] = 0
      return out
  elif noise_typ == "poisson":
      vals = len(np.unique(image))
      vals = 2 ** np.ceil(np.log2(vals))
      noisy = np.random.poisson(image * vals) / float(vals)
      return noisy
  elif noise_typ =="speckle":
      row,col,ch = image.shape
      gauss = np.random.randn(row,col,ch)
      gauss = gauss.reshape(row,col,ch)        
      noisy = image + image * gauss
      return noisy

This view is not constrained vertically. At runtime it will jump to the left unless you add a vertical constraint

You need to drag the EditText from the edge of the layout and not just the other widget. You can also add constraints by just dragging the constraint point that surrounds the widget to the edge of the screen to add constraints as specified.

The modified code will look something similar to this:

    app:layout_constraintLeft_toLeftOf="@+id/router_text"
    app:layout_constraintTop_toTopOf="@+id/activity_main"
    android:layout_marginTop="320dp"
    app:layout_constraintBottom_toBottomOf="@+id/activity_main"
    android:layout_marginBottom="16dp"
    app:layout_constraintVertical_bias="0.29" 

Collection was modified; enumeration operation may not execute in ArrayList

One way is to add the item(s) to be deleted to a new list. Then go through and delete those items.

How do I create a simple Qt console application in C++?

I managed to create a simple console "hello world" with QT Creator

used creator 2.4.1 and QT 4.8.0 on windows 7

two ways to do this

Plain C++

do the following

  1. File- new file project
  2. under projects select : other Project
  3. select "Plain C++ Project"
  4. enter project name 5.Targets select Desktop 'tick it'
  5. project managment just click next
  6. you can use c++ commands as normal c++

or

QT Console

  1. File- new file project
  2. under projects select : other Project
  3. select QT Console Application
  4. Targets select Desktop 'tick it'
  5. project managment just click next
  6. add the following lines (all the C++ includes you need)
  7. add "#include 'iostream' "
  8. add "using namespace std; "
  9. after QCoreApplication a(int argc, cghar *argv[]) 10 add variables, and your program code..

example: for QT console "hello world"

file - new file project 'project name '

other projects - QT Console Application

Targets select 'Desktop'

project management - next

code:

    #include <QtCore/QCoreApplication>
    #include <iostream>
    using namespace std;
    int main(int argc, char *argv[])
    {
     QCoreApplication a(argc, argv);
     cout<<" hello world";
     return a.exec();
     }

ctrl -R to run

compilers used for above MSVC 2010 (QT SDK) , and minGW(QT SDK)

hope this helps someone

As I have just started to use QT recently and also searched the Www for info and examples to get started with simple examples still searching...

Trying to handle "back" navigation button action in iOS

None of the other solutions worked for me, but this does:

Create your own subclass of UINavigationController, make it implement the UINavigationBarDelegate (no need to manually set the navigation bar's delegate), add a UIViewController extension that defines a method to be called on a back button press, and then implement this method in your UINavigationController subclass:

func navigationBar(_ navigationBar: UINavigationBar, shouldPop item: UINavigationItem) -> Bool {
    self.topViewController?.methodToBeCalledOnBackButtonPress()
    self.popViewController(animated: true)
    return true
}

How to calculate a time difference in C++

If you are using:

tstart = clock();

// ...do something...

tend = clock();

Then you will need the following to get time in seconds:

time = (tend - tstart) / (double) CLOCKS_PER_SEC;

How to use opencv in using Gradle?

OpenCV, Android Studio 1.4.1, gradle-experimental plugin 0.2.1

None of the other answers helped me. Here's what worked for me. I'm using the tutorial-1 sample from opencv but I will be doing using the NDK in my project so I'm using the gradle-experimental plugin which has a different structure than the gradle plugin.

Android studio should be installed, the Android NDK should be installed via the Android SDK Manager, and the OpenCV Android SDK should be downloaded and unzipped.

This is in chunks of bash script to keep it compact but complete. It's also all on the command line because on of the big problems I had was that in-IDE instructions were obsolete as the IDE evolved.

First set the location of the root directory of the OpenCV SDK.

export OPENCV_SDK=/home/user/wip/OpenCV-2.4.11-android-sdk
cd $OPENCV_SDK

Create your gradle build files...

First the OpenCV library

cat > $OPENCV_SDK/sdk/java/build.gradle <<'==='


apply plugin: 'com.android.model.library'
model {
    android {
        compileSdkVersion = 23
        buildToolsVersion = "23.0.2"

        defaultConfig.with {
            minSdkVersion.apiLevel = 8
            targetSdkVersion.apiLevel = 23
        }
    }

    android.buildTypes {
        release {
            minifyEnabled = false
        }
        debug{
            minifyEnabled = false
        }
    }
   android.sources {
       main.manifest.source.srcDirs +=  "."
       main.res.source.srcDirs +=  "res"
       main.aidl.source.srcDirs +=  "src"
       main.java.source.srcDirs +=  "src"
   }

}


===

Then tell the tutorial sample what to label the library as and where to find it.

cat > $OPENCV_SDK/samples/tutorial-1-camerapreview/settings.gradle <<'==='


include ':openCVLibrary2411'
project(':openCVLibrary2411').projectDir = new File('../../sdk/java')


===

Create the build file for the tutorial.

cat > $OPENCV_SDK/samples/tutorial-1-camerapreview/build.gradle <<'==='


buildscript {
    repositories {
        jcenter()
    }
    dependencies {
        classpath 'com.android.tools.build:gradle-experimental:0.2.1'
    }
}

allprojects {
    repositories {
        jcenter()
    }
}

apply plugin: 'com.android.model.application'

model {
    android {
        compileSdkVersion = 23
        buildToolsVersion = "23.0.2"

        defaultConfig.with {
            applicationId = "org.opencv.samples.tutorial1"
            minSdkVersion.apiLevel = 8
            targetSdkVersion.apiLevel = 23
        }
    }

    android.sources {
       main.manifest.source.srcDirs +=  "."
       main.res.source.srcDirs +=  "res"
       main.aidl.source.srcDirs +=  "src"
       main.java.source.srcDirs +=  "src"
    } 

    android.buildTypes {
        release {
            minifyEnabled = false
            proguardFiles += file('proguard-rules.pro')
        }
        debug {
             minifyEnabled = false
        }
    }
}

dependencies {
    compile project(':openCVLibrary2411')
}


===

Your build tools version needs to be set correctly. Here's an easy way to see what you have installed. (You can install other versions via the Android SDK Manager). Change buildToolsVersion if you don't have 23.0.2.

echo "Your buildToolsVersion is one of: "
ls $ANDROID_HOME/build-tools

Change the environment variable on the first line to your version number

REP=23.0.2 #CHANGE ME
sed -i.bak s/23\.0\.2/${REP}/g $OPENCV_SDK/sdk/java/build.gradle
sed -i.bak s/23\.0\.2/${REP}/g $OPENCV_SDK/samples/tutorial-1-camerapreview/build.gradle

Finally, set up the correct gradle wrapper. Gradle needs a clean directory to do this.

pushd $(mktemp -d)

gradle wrapper --gradle-version 2.5

mv -f gradle* $OPENCV_SDK/samples/tutorial-1-camerapreview
popd

You should now be all set. You can now browse to this directory with Android Studio and open up the project.

Build the tutoral on the command line with the following command:

./gradlew assembleDebug

It should build your apk, putting it in ./build/outputs/apk

How to stop PHP code execution?

You can use __halt_compiler function which will Halt the compiler execution

http://www.php.net/manual/en/function.halt-compiler.php

How to fix: "UnicodeDecodeError: 'ascii' codec can't decode byte"

I find the best is to always convert to unicode - but this is difficult to achieve because in practice you'd have to check and convert every argument to every function and method you ever write that includes some form of string processing.

So I came up with the following approach to either guarantee unicodes or byte strings, from either input. In short, include and use the following lambdas:

# guarantee unicode string
_u = lambda t: t.decode('UTF-8', 'replace') if isinstance(t, str) else t
_uu = lambda *tt: tuple(_u(t) for t in tt) 
# guarantee byte string in UTF8 encoding
_u8 = lambda t: t.encode('UTF-8', 'replace') if isinstance(t, unicode) else t
_uu8 = lambda *tt: tuple(_u8(t) for t in tt)

Examples:

text='Some string with codes > 127, like Zürich'
utext=u'Some string with codes > 127, like Zürich'
print "==> with _u, _uu"
print _u(text), type(_u(text))
print _u(utext), type(_u(utext))
print _uu(text, utext), type(_uu(text, utext))
print "==> with u8, uu8"
print _u8(text), type(_u8(text))
print _u8(utext), type(_u8(utext))
print _uu8(text, utext), type(_uu8(text, utext))
# with % formatting, always use _u() and _uu()
print "Some unknown input %s" % _u(text)
print "Multiple inputs %s, %s" % _uu(text, text)
# but with string.format be sure to always work with unicode strings
print u"Also works with formats: {}".format(_u(text))
print u"Also works with formats: {},{}".format(*_uu(text, text))
# ... or use _u8 and _uu8, because string.format expects byte strings
print "Also works with formats: {}".format(_u8(text))
print "Also works with formats: {},{}".format(*_uu8(text, text))

Here's some more reasoning about this.

Fixed page header overlaps in-page anchors

For Chrome/Safari/Firefox you could add a display: block and use a negative margin to compensate the offset, like:

a[name] {
    display: block;
    padding-top: 90px;
    margin-top: -90px;
}

See example http://codepen.io/swed/pen/RrZBJo

How do I kill a process using Vb.NET or C#?

I opened one Word file, 2. Now I open another word file through vb.net runtime programmatically. 3. I want to kill the second process alone through programmatically. 4. Do not kill first process

Using FolderBrowserDialog in WPF application

You need to add a reference to System.Windows.Forms.dll, then use the System.Windows.Forms.FolderBrowserDialog class.

Adding using WinForms = System.Windows.Forms; will be helpful.

Calculate number of hours between 2 dates in PHP

This function helps you to calculate exact years and months between two given dates, $doj1 and $doj. It returns example 4.3 means 4 years and 3 month.

<?php
    function cal_exp($doj1)
    {
        $doj1=strtotime($doj1);
        $doj=date("m/d/Y",$doj1); //till date or any given date

        $now=date("m/d/Y");
        //$b=strtotime($b1);
        //echo $c=$b1-$a2;
        //echo date("Y-m-d H:i:s",$c);
        $year=date("Y");
        //$chk_leap=is_leapyear($year);

        //$year_diff=365.25;

        $x=explode("/",$doj);
        $y1=explode("/",$now);

        $yy=$x[2];
        $mm=$x[0];
        $dd=$x[1];

        $yy1=$y1[2];
        $mm1=$y1[0];
        $dd1=$y1[1];
        $mn=0;
        $mn1=0;
        $ye=0;
        if($mm1>$mm)
        {
            $mn=$mm1-$mm;
            if($dd1<$dd)
            {
                $mn=$mn-1;
            }
            $ye=$yy1-$yy;
        }
        else if($mm1<$mm)
        {
            $mn=12-$mm;
            //$mn=$mn;

            if($mm!=1)
            {
                $mn1=$mm1-1;
            }

            $mn+=$mn1;
            if($dd1>$dd)
            {
                $mn+=1;
            }

            $yy=$yy+1;
            $ye=$yy1-$yy;
        }
        else
        {
            $ye=$yy1-$yy;
            $ye=$ye-1;

            $mn=12-1;

            if($dd1>$dd)
            {
                $ye+=1;
                $mn=0;
            }
        }

        $to=$ye." year and ".$mn." months";
        return $ye.".".$mn;

        /*return daysDiff($x[2],$x[0],$x[1]);
         $days=dateDiff("/",$now,$doj)/$year_diff;
        $days_exp=explode(".",$days);
        return $years_exp=$days; //number of years exp*/
    }
?>

Sorting Python list based on the length of the string

Write a function lensort to sort a list of strings based on length.

def lensort(a):
    n = len(a)
    for i in range(n):
        for j in range(i+1,n):
            if len(a[i]) > len(a[j]):
                temp = a[i]
                a[i] = a[j]
                a[j] = temp
    return a
print lensort(["hello","bye","good"])

pandas dataframe groupby datetime month

One solution which avoids MultiIndex is to create a new datetime column setting day = 1. Then group by this column.

Normalise day of month

df = pd.DataFrame({'Date': pd.to_datetime(['2017-10-05', '2017-10-20', '2017-10-01', '2017-09-01']),
                   'Values': [5, 10, 15, 20]})

# normalize day to beginning of month, 4 alternative methods below
df['YearMonth'] = df['Date'] + pd.offsets.MonthEnd(-1) + pd.offsets.Day(1)
df['YearMonth'] = df['Date'] - pd.to_timedelta(df['Date'].dt.day-1, unit='D')
df['YearMonth'] = df['Date'].map(lambda dt: dt.replace(day=1))
df['YearMonth'] = df['Date'].dt.normalize().map(pd.tseries.offsets.MonthBegin().rollback)

Then use groupby as normal:

g = df.groupby('YearMonth')

res = g['Values'].sum()

# YearMonth
# 2017-09-01    20
# 2017-10-01    30
# Name: Values, dtype: int64

Comparison with pd.Grouper

The subtle benefit of this solution is, unlike pd.Grouper, the grouper index is normalized to the beginning of each month rather than the end, and therefore you can easily extract groups via get_group:

some_group = g.get_group('2017-10-01')

Calculating the last day of October is slightly more cumbersome. pd.Grouper, as of v0.23, does support a convention parameter, but this is only applicable for a PeriodIndex grouper.

Comparison with string conversion

An alternative to the above idea is to convert to a string, e.g. convert datetime 2017-10-XX to string '2017-10'. However, this is not recommended since you lose all the efficiency benefits of a datetime series (stored internally as numerical data in a contiguous memory block) versus an object series of strings (stored as an array of pointers).

When should I use File.separator and when File.pathSeparator?

java.io.File class contains four static separator variables. For better understanding, Let's understand with the help of some code

  1. separator: Platform dependent default name-separator character as String. For windows, it’s ‘\’ and for unix it’s ‘/’
  2. separatorChar: Same as separator but it’s char
  3. pathSeparator: Platform dependent variable for path-separator. For example PATH or CLASSPATH variable list of paths separated by ‘:’ in Unix systems and ‘;’ in Windows system
  4. pathSeparatorChar: Same as pathSeparator but it’s char

Note that all of these are final variables and system dependent.

Here is the java program to print these separator variables. FileSeparator.java

import java.io.File;

public class FileSeparator {

    public static void main(String[] args) {
        System.out.println("File.separator = "+File.separator);
        System.out.println("File.separatorChar = "+File.separatorChar);
        System.out.println("File.pathSeparator = "+File.pathSeparator);
        System.out.println("File.pathSeparatorChar = "+File.pathSeparatorChar);
    }

}

Output of above program on Unix system:

File.separator = /
File.separatorChar = /
File.pathSeparator = :
File.pathSeparatorChar = :

Output of the program on Windows system:

File.separator = \
File.separatorChar = \
File.pathSeparator = ;
File.pathSeparatorChar = ;

To make our program platform independent, we should always use these separators to create file path or read any system variables like PATH, CLASSPATH.

Here is the code snippet showing how to use separators correctly.

//no platform independence, good for Unix systems
File fileUnsafe = new File("tmp/abc.txt");
//platform independent and safe to use across Unix and Windows
File fileSafe = new File("tmp"+File.separator+"abc.txt");

Nginx: stat() failed (13: permission denied)

This is how i fixed this

sudo chmod o+x /home/ec2-user

VBA Subscript out of range - error 9

Suggest the following simplification: capture return value from Workbooks.Add instead of subscripting Windows() afterward, as follows:

Set wkb = Workbooks.Add
wkb.SaveAs ...

wkb.Activate ' instead of Windows(expression).Activate


General Philosophy Advice:

Avoid use Excel's built-ins: ActiveWorkbook, ActiveSheet, and Selection: capture return values, and, favor qualified expressions instead.

Use the built-ins only once and only in outermost macros(subs) and capture at macro start, e.g.

Set wkb = ActiveWorkbook
Set wks = ActiveSheet
Set sel = Selection

During and within macros do not rely on these built-in names, instead capture return values, e.g.

Set wkb = Workbooks.Add 'instead of Workbooks.Add without return value capture
wkb.Activate 'instead of Activeworkbook.Activate

Also, try to use qualified expressions, e.g.

wkb.Sheets("Sheet3").Name = "foo" ' instead of Sheets("Sheet3").Name = "foo"

or

Set newWks = wkb.Sheets.Add
newWks.Name = "bar" 'instead of ActiveSheet.Name = "bar"

Use qualified expressions, e.g.

newWks.Name = "bar" 'instead of `xyz.Select` followed by Selection.Name = "bar" 

These methods will work better in general, give less confusing results, will be more robust when refactoring (e.g. moving lines of code around within and between methods) and, will work better across versions of Excel. Selection, for example, changes differently during macro execution from one version of Excel to another.

Also please note that you'll likely find that you don't need to .Activate nearly as much when using more qualified expressions. (This can mean the for the user the screen will flicker less.) Thus the whole line Windows(expression).Activate could simply be eliminated instead of even being replaced by wkb.Activate.

(Also note: I think the .Select statements you show are not contributing and can be omitted.)

(I think that Excel's macro recorder is responsible for promoting this more fragile style of programming using ActiveSheet, ActiveWorkbook, Selection, and Select so much; this style leaves a lot of room for improvement.)

Android ClassNotFoundException: Didn't find class on path

I had this problem after upgrading from version 18. After the upgrade I had the following: Project Build Target was 5.01, targetSDKVersion=21

But: Build Tools were still from Android 4.3.1 (18) What I noticed was that those apps that were deriving their activities from ActionBarActivity crashed with the above error those that did not still ran.

So the simple fix was to use the latest Android SDK Build Tools 21.1.2 instead of only those of Android SDK Build Tools 18, which seems somehow the default.

So the culprit seems the Appcompat library, which I use for Actionbar backward compability.

And a by the way: using the appcompat library I had to set/fix: $SDKInstallDIR$\Android\android-sdk\extras\android\support\v7\appcompat\project.properties target=android-21 (which was recommended in some other useful post and which has a value of 19 by default))

How to make jQuery UI nav menu horizontal?

I know this is an old thread but I was digging into the jQuery UI source code and built upon Rowan's answer which was closer to what I had been looking for. Only I needed the carrots as I felt it was important to have a visual indicator of possible submenus. From looking at the source code (both .js and .css) I came up with this that allows the carrot be visiable without messing with design (height) and also alowing menu to appear vertical below the parent item.

In the jquery-ui.js do a search to find $.widget( "ui.menu") and change postition to:

position: {
my: "center top",
at: "center bottom"
}

And in your css add:

#nav > .ui-menu:after {
    content: ".";
    display: block;
    clear: both;
    visibility: hidden;
    line-height: 0;
    height: 0;
}

#nav > .ui-menu > .ui-menu-item {
    display: inline-block;
    float: left;

    margin: 0;
    padding: 3px;

    width: auto;
}

.ui-menu .ui-menu-item a {
    padding: 0;
}

.ui-menu .ui-menu-icon{
    position: relative;
    left: 1px;
    top: 6px;
}

End Result will be a jQuery UI Menu horizontal with sub menus being displayed veriticaly below the parent menu item.

Checking cin input stream produces an integer

There is a function in c called isdigit(). That will suit you just fine. Example:

int var1 = 'h';
int var2 = '2';

if( isdigit(var1) )
{
   printf("var1 = |%c| is a digit\n", var1 );
}
else
{
   printf("var1 = |%c| is not a digit\n", var1 );
}
if( isdigit(var2) )
{
  printf("var2 = |%c| is a digit\n", var2 );
}
else
{
   printf("var2 = |%c| is not a digit\n", var2 );
}

From here

Loading existing .html file with android WebView

paste your .html file in assets folder of your project folder. and create an xml file in layout folder with the fol code: my.xml:

<WebView  xmlns:android="http://schemas.android.com/apk/res/android"
  android:id="@+id/webview"
  android:layout_width="fill_parent"
  android:layout_height="fill_parent"
    />

add fol code in activity

setContentView(R.layout.my);
    WebView mWebView = null;
    mWebView = (WebView) findViewById(R.id.webview);
    mWebView.getSettings().setJavaScriptEnabled(true);
    mWebView.loadUrl("file:///android_asset/new.html"); //new.html is html file name.

Setting up maven dependency for SQL Server

Be careful with the answers above. sqljdbc4.jar is not distributed with under a public license which is why it is difficult to include it in a jar for runtime and distribution. See my answer below for more details and a much better solution. Your life will become much easier as mine did once I found this answer.

https://stackoverflow.com/a/30111956/3368958

Error: package or namespace load failed for ggplot2 and for data.table

Try this:

install.packages('Rcpp')
install.packages('ggplot2')
install.packages('data.table')

Read Post Data submitted to ASP.Net Form

Read the Request.Form NameValueCollection and process your logic accordingly:

NameValueCollection nvc = Request.Form;
string userName, password;
if (!string.IsNullOrEmpty(nvc["txtUserName"]))
{
  userName = nvc["txtUserName"];
}

if (!string.IsNullOrEmpty(nvc["txtPassword"]))
{
  password = nvc["txtPassword"];
}

//Process login
CheckLogin(userName, password);

... where "txtUserName" and "txtPassword" are the Names of the controls on the posting page.

"Could not find bundler" error

If you're using rbenv running rbenv rehash can solve this after you've installed bundler and are still getting the issue.

Javascript Array Alert

If you want to see the array as an array, you can say

alert(JSON.stringify(aCustomers));

instead of all those document.writes.

http://jsfiddle.net/5b2eb/

However, if you want to display them cleanly, one per line, in your popup, do this:

alert(aCustomers.join("\n"));

http://jsfiddle.net/5b2eb/1/

Does reading an entire file leave the file handle open?

You can use pathlib.

For Python 3.5 and above:

from pathlib import Path
contents = Path(file_path).read_text()

For older versions of Python use pathlib2:

$ pip install pathlib2

Then:

from pathlib2 import Path
contents = Path(file_path).read_text()

This is the actual read_text implementation:

def read_text(self, encoding=None, errors=None):
    """
    Open the file in text mode, read it, and close the file.
    """
    with self.open(mode='r', encoding=encoding, errors=errors) as f:
        return f.read()

Javascript (+) sign concatenates instead of giving sum of variables

Since you are concatenating numbers on to a string, the whole thing is treated as a string. When you want to add numbers together, you either need to do it separately and assign it to a var and use that var, like this:

i = i + 1;
divID = "question-" + i;

Or you need to specify the number addition like this:

divID = "question-" + Number(i+1);

EDIT

I should have added this long ago, but based on the comments, this works as well:

divID = "question-" + (i+1);

CSS Grid Layout not working in IE11 even with prefixes

To support IE11 with auto-placement, I converted grid to table layout every time I used the grid layout in 1 dimension only. I also used margin instead of grid-gap.

The result is the same, see how you can do it here https://jsfiddle.net/hp95z6v1/3/

How to override trait function and call it from the overridden function?

If the class implements the method directly, it will not use the traits version. Perhaps what you are thinking of is:

trait A {
    function calc($v) {
        return $v+1;
    }
}

class MyClass {
    function calc($v) {
        return $v+2;
    }
}

class MyChildClass extends MyClass{
}

class MyTraitChildClass extends MyClass{
    use A;
}

print (new MyChildClass())->calc(2); // will print 4

print (new MyTraitChildClass())->calc(2); // will print 3

Because the child classes do not implement the method directly, they will first use that of the trait if there otherwise use that of the parent class.

If you want, the trait can use method in the parent class (assuming you know the method would be there) e.g.

trait A {
    function calc($v) {
        return parent::calc($v*3);
    }
}
// .... other code from above
print (new MyTraitChildClass())->calc(2); // will print 8 (2*3 + 2)

You can also provide for ways to override, but still access the trait method as follows:

trait A {
    function trait_calc($v) {
        return $v*3;
    }
}

class MyClass {
    function calc($v) {
        return $v+2;
    }
}


class MyTraitChildClass extends MyClass{
    use A {
      A::trait_calc as calc;
    }
}


class MySecondTraitChildClass extends MyClass{
    use A {
      A::trait_calc as calc;
    }

    public function calc($v) {
      return $this->trait_calc($v)+.5;
    }
}


print (new MyTraitChildClass())->calc(2); // will print 6
echo "\n";
print (new MySecondTraitChildClass())->calc(2); // will print 6.5

You can see it work at http://sandbox.onlinephpfunctions.com/code/e53f6e8f9834aea5e038aec4766ac7e1c19cc2b5

Characters allowed in GET parameter

There are reserved characters, that have a reserved meanings, those are delimiters — :/?#[]@ — and subdelimiters — !$&'()*+,;=

There is also a set of characters called unreserved characters — alphanumerics and -._~ — which are not to be encoded.

That means, that anything that doesn't belong to unreserved characters set is supposed to be %-encoded, when they do not have special meaning (e.g. when passed as a part of GET parameter).

See also RFC3986: Uniform Resource Identifier (URI): Generic Syntax

What is the difference between single and double quotes in SQL?

The difference lies in their usage. The single quotes are mostly used to refer a string in WHERE, HAVING and also in some built-in SQL functions like CONCAT, STRPOS, POSITION etc.

When you want to use an alias that has space in between then you can use double quotes to refer to that alias.

For example

(select account_id,count(*) "count of" from orders group by 1)sub 

Here is a subquery from an orders table having account_id as Foreign key that I am aggregating to know how many orders each account placed. Here I have given one column any random name as "count of" for sake of purpose.

Now let's write an outer query to display the rows where "count of" is greater than 20.

select "count of" from 
(select account_id,count(*) "count of" from orders group by 1)sub where "count of" >20;

You can apply the same case to Common Table expressions also.

Git: Recover deleted (remote) branch

Your deleted branches are not lost, they were copied into origin/contact_page and origin/new_pictures “remote tracking branches” by the fetch you showed (they were also pushed back out by the push you showed, but they were pushed into refs/remotes/origin/ instead of refs/heads/). Check git log origin/contact_page and git log origin/new_pictures to see if your local copies are “up to date” with whatever you think should be there. If any new commits were pushed onto those branches (from some other repo) between the fetch and push that you showed, you may have “lost” those (but probably you could probably find them in the other repo that most recently pushed those branches).

Fetch/Push Conflict

It looks like you are fetching in a normal, ‘remote mode’ (remote refs/heads/ are stored locally in refs/remotes/origin/), but pushing in ‘mirror mode’ (local refs/ are pushed onto remote refs/). Check your .git/config and reconcile the remote.origin.fetch and remote.origin.push settings.

Make a Backup

Before trying any changes, make a simple tar or zip archive or your whole local repo. That way, if you do not like what happens, you can try again from a restored repo.

Option A: Reconfigure as a Mirror

If you intend to use your remote repo as a mirror of your local one, do this:

git branch contact_page origin/contact_page &&
git branch new_pictures origin/new_pictures &&
git config remote.origin.fetch '+refs/*:refs/*' &&
git config --unset remote.origin.push &&
git config remote.origin.mirror true

You might also eventually want to do delete all your refs/remotes/origin/ refs, since they are not useful if you are operating in mirror mode (your normal branches take the place of the usual remote tracking branches).

Option B: Reconfigure as a Normal Remote

But since it seems that you are using this remote repo with multiple “work” repos, you probably do not want to use mirror mode. You might try this:

git config push.default tracking &&
git config --unset remote.origin.push
git config --unset remote.origin.mirror

Then, you will eventually want to delete the bogus refs/remotes/origin refs in your remote repo: git push origin :refs/remotes/origin/contact_page :refs/remotes/origin/new_pictures ….

Test Push

Try git push --dry-run to see what it git push would do without having it make any changes on the remote repo. If you do not like what it says it is going to do, recover from your backup (tar/zip) and try the other option.

jQuery select option elements by value

options = $("#span_id>select>option[value='"+i+"']");
option = options.text();
alert(option); 

here is the fiddle http://jsfiddle.net/hRFYF/

Receiving "fatal: Not a git repository" when attempting to remote add a Git repo

Did you init a local Git repository, into which this remote is supposed to be added?

Does your local directory have a .git folder?

Try git init.

Convert an image (selected by path) to base64 string

You can convert it like this

  string test = @"C:/image/1.gif";
  byte[] bytes = System.Text.ASCIIEncoding.ASCII.GetBytes(test);
  string base64String = System.Convert.ToBase64String(bytes);
  Console.WriteLine("Base 64 string: " + base64String);

Output

  QzovaW1hZ2UvMS5naWY=

How to determine day of week by passing specific date?

You can use below method to get Day of the Week by passing a specific date,

Here for the set method of Calendar class, Tricky part is the index for the month parameter will starts from 0.

public static String getDay(int day, int month, int year) {

        Calendar cal = Calendar.getInstance();

        if(month==1){
            cal.set(year,0,day);
        }else{
            cal.set(year,month-1,day);
        }

        int dow = cal.get(Calendar.DAY_OF_WEEK);

        switch (dow) {
        case 1:
            return "SUNDAY";
        case 2:
            return "MONDAY";
        case 3:
            return "TUESDAY";
        case 4:
            return "WEDNESDAY";
        case 5:
            return "THURSDAY";
        case 6:
            return "FRIDAY";
        case 7:
            return "SATURDAY";
        default:
            System.out.println("GO To Hell....");
        }

        return null;
    }

How to find all positions of the maximum value in a list?

If you want to get the indices of the largest n numbers in a list called data, you can use Pandas sort_values:

pd.Series(data).sort_values(ascending=False).index[0:n]

Why can't non-default arguments follow default arguments?

All required parameters must be placed before any default arguments. Simply because they are mandatory, whereas default arguments are not. Syntactically, it would be impossible for the interpreter to decide which values match which arguments if mixed modes were allowed. A SyntaxError is raised if the arguments are not given in the correct order:

Let us take a look at keyword arguments, using your function.

def fun1(a="who is you", b="True", x, y):
...     print a,b,x,y

Suppose its allowed to declare function as above, Then with the above declarations, we can make the following (regular) positional or keyword argument calls:

func1("ok a", "ok b", 1)  # Is 1 assigned to x or ?
func1(1)                  # Is 1 assigned to a or ?
func1(1, 2)               # ?

How you will suggest the assignment of variables in the function call, how default arguments are going to be used along with keyword arguments.

>>> def fun1(x, y, a="who is you", b="True"):
...     print a,b,x,y
... 

Reference O'Reilly - Core-Python
Where as this function make use of the default arguments syntactically correct for above function calls. Keyword arguments calling prove useful for being able to provide for out-of-order positional arguments, but, coupled with default arguments, they can also be used to "skip over" missing arguments as well.

MySQL: update a field only if condition is met

Yes!

Here you have another example:

UPDATE prices
SET final_price= CASE
   WHEN currency=1 THEN 0.81*final_price
   ELSE final_price
END

This works because MySQL doesn't update the row, if there is no change, as mentioned in docs:

If you set a column to the value it currently has, MySQL notices this and does not update it.

How to hide a div after some time period?

In older versions of jquery you'll have to do it the "javascript way" using settimeout

setTimeout( function(){$('div').hide();} , 4000);

or

setTimeout( "$('div').hide();", 4000);

Recently with jquery 1.4 this solution has been added:

$("div").delay(4000).hide();

Of course replace "div" by the correct element using a valid jquery selector and call the function when the document is ready.

ArrayAdapter in android to create simple listview

public ArrayAdapter (Context context, int resource, int textViewResourceId, T[] objects)

I am also new to Android , so i might be wrong. But as per my understanding while using this for listview creation 2nd argument is the layout of list items. A layout consists of many views (image view,text view etc). With 3rd argument you are specifying in which view or textview you want the text to be displayed.

Do you have to include <link rel="icon" href="favicon.ico" type="image/x-icon" />?

Many people set their cookie path to /. That will cause every favicon request to send a copy of the sites cookies, at least in chrome. Addressing your favicon to your cookieless domain should correct this.

<link rel="icon" href="https://cookieless.MySite.com/favicon.ico" type="image/x-icon" />

Depending on how much traffic you get, this may be the most practical reason for adding the link.

Info on setting up a cookieless domain:

http://www.ravelrumba.com/blog/static-cookieless-domain/

Hide "NFC Tag type not supported" error on Samsung Galaxy devices

Before Android 4.4

What you are trying to do is simply not possible from an app (at least not on a non-rooted/non-modified device). The message "NFC tag type not supported" is displayed by the Android system (or more specifically the NFC system service) before and instead of dispatching the tag to your app. This means that the NFC system service filters MIFARE Classic tags and never notifies any app about them. Consequently, your app can't detect MIFARE Classic tags or circumvent that popup message.

On a rooted device, you may be able to bypass the message using either

  1. Xposed to modify the behavior of the NFC service, or
  2. the CSC (Consumer Software Customization) feature configuration files on the system partition (see /system/csc/. The NFC system service disables the popup and dispatches MIFARE Classic tags to apps if the CSC feature <CscFeature_NFC_EnableSecurityPromptPopup> is set to any value but "mifareclassic" or "all". For instance, you could use:

    <CscFeature_NFC_EnableSecurityPromptPopup>NONE</CscFeature_NFC_EnableSecurityPromptPopup>
    

    You could add this entry to, for instance, the file "/system/csc/others.xml" (within the section <FeatureSet> ... </FeatureSet> that already exists in that file).

Since, you asked for the Galaxy S6 (the question that you linked) as well: I have tested this method on the S4 when it came out. I have not verified if this still works in the latest firmware or on other devices (e.g. the S6).

Since Android 4.4

This is pure guessing, but according to this (link no longer available), it seems that some apps (e.g. NXP TagInfo) are capable of detecting MIFARE Classic tags on affected Samsung devices since Android 4.4. This might mean that foreground apps are capable of bypassing that popup using the reader-mode API (see NfcAdapter.enableReaderMode) possibly in combination with NfcAdapter.FLAG_READER_SKIP_NDEF_CHECK.

Which is the correct C# infinite loop, for (;;) or while (true)?

If you're code-golfing, I would suggest for(;;). Beyond that, while(true) has the same meaning and seems more intuitive. At any rate, most coders will likely understand both variations, so it doesn't really matter. Use what's most comfortable.

Connecting client to server using Socket.io

You need to make sure that you add forward slash before your link to socket.io:

<script src="/socket.io/socket.io.js"></script>

Then in the view/controller just do:

var socket = io.connect()

That should solve your problem.

calling server side event from html button control

You may use event handler serverclick as below

//cmdAction is the id of HTML button as below

<body>
    <form id="form1" runat="server">
        <button type="submit" id="cmdAction" text="Button1" runat="server">
            Button1
        </button>
    </form>
</body>

//cs code

public partial class _Default : System.Web.UI.Page
    {
        protected void Page_Load(object sender, EventArgs e)
        {
                cmdAction.ServerClick += new EventHandler(submit_click);  
        }

        protected void submit_click(object sender, EventArgs e)
        {
            Response.Write("HTML Server Button Control");
        }
    }

How to get the current location latitude and longitude in android

**The activity should implements LocationListener

In onCreate(), write the following code **

  Boolean network = haveNetworkConnection();
    Log.e("network", "---------->" + network);
    if (!network) {
        Toast.makeText(getApplicationContext(), "Network is not available",
                3000).show();

    }

    SupportMapFragment supportMapFragment = (SupportMapFragment) getSupportFragmentManager()
            .findFragmentById(R.id.googleMap);
    googleMap = supportMapFragment.getMap();
    googleMap.setMyLocationEnabled(true);


    LocationManager locationManager = (LocationManager) getSystemService(LOCATION_SERVICE);
    locationManager.requestLocationUpdates(LocationManager.GPS_PROVIDER, 30000, 0, this);




        if (!locationManager.isProviderEnabled(LocationManager.GPS_PROVIDER)
            && !locationManager
                    .isProviderEnabled(LocationManager.NETWORK_PROVIDER)) {



         TextView title = new TextView(context);
         title.setText("Location Services Not Active");
            title.setBackgroundColor(Color.BLACK);
            title.setPadding(10, 15, 15, 10);
            title.setGravity(Gravity.CENTER);
            title.setTextColor(Color.WHITE);
            title.setTextSize(22);


            AlertDialog.Builder builder = new AlertDialog.Builder(this);




        builder.setCustomTitle(title);


        // builder.setTitle("Location Services Not Active");
        builder.setMessage("Please enable Location Services and GPS");

        builder.setPositiveButton("Turn on",
                new DialogInterface.OnClickListener() {
                    public void onClick(DialogInterface dialogInterface,
                            int i) {
                        // Show location settings when the user acknowledges
                        // the alert dialog
                        Intent intent = new Intent(
                                Settings.ACTION_LOCATION_SOURCE_SETTINGS);

                        startActivity(intent);
                        finish();
                    }
                });

        builder.setNegativeButton("Cancel",
                new DialogInterface.OnClickListener() {

                    @Override
                    public void onClick(DialogInterface dialog, int which) {
                        // TODO Auto-generated method stub
                        dialog.cancel();
                    }
                });

        builder.show();

    }

    Criteria criteria = new Criteria();
    String bestProvider = locationManager.getBestProvider(criteria, true);
    Location location = locationManager.getLastKnownLocation(bestProvider);

    if (location == null) {
        Toast.makeText(getApplicationContext(), "GPS signal not found",
                3000).show();
    }
    if (location != null) {
        Log.e("locatin", "location--" + location);

        Log.e("latitude at beginning",
                "@@@@@@@@@@@@@@@" + location.getLatitude());
        onLocationChanged(location);
    }

Write a method haveNetworkConnection

private boolean haveNetworkConnection() {
    boolean haveConnectedWifi = false;
    boolean haveConnectedMobile = false;

    ConnectivityManager cm = (ConnectivityManager) getSystemService(Context.CONNECTIVITY_SERVICE);
    NetworkInfo[] netInfo = cm.getAllNetworkInfo();
    for (NetworkInfo ni : netInfo) {
        if (ni.getTypeName().equalsIgnoreCase("WIFI"))
            if (ni.isConnected())
                haveConnectedWifi = true;
        if (ni.getTypeName().equalsIgnoreCase("MOBILE"))
            if (ni.isConnected())
                haveConnectedMobile = true;
    }
    return haveConnectedWifi || haveConnectedMobile;
}



@Override
public void onLocationChanged(Location location) {


    LatLng latLng = new LatLng(latitude, longitude);
    googleMap.addMarker(new MarkerOptions()
            .position(latLng)
            .title("Current LOC")
            .icon(BitmapDescriptorFactory
                    .defaultMarker(BitmapDescriptorFactory.HUE_RED)));

    googleMap.moveCamera(CameraUpdateFactory.newLatLng(latLng));
    googleMap.animateCamera(CameraUpdateFactory.zoomTo(17));


}

@Override
public void onProviderDisabled(String provider) {
    // TODO Auto-generated method stub
}

@Override
public void onProviderEnabled(String provider) {
    // TODO Auto-generated method stub
}

@Override
public void onStatusChanged(String provider, int status, Bundle extras) {
    // TODO Auto-generated method stub
}

How to rename a pane in tmux?

You can adjust the pane title by setting the pane border in the tmux.conf for example like this:

###############
# pane border #
###############
set -g pane-border-status bottom
#colors for pane borders
setw -g pane-border-style fg=green,bg=black
setw -g pane-active-border-style fg=colour118,bg=black
setw -g automatic-rename off
setw -g pane-border-format ' #{pane_index} #{pane_title} : #{pane_current_path} '
# active pane normal, other shaded out?
setw -g window-style fg=colour28,bg=colour16
setw -g window-active-style fg=colour46,bg=colour16

Where pane_index, pane_title and pane_current_path are variables provided by tmux itself.

After reloading the config or starting a new tmux session, you can then set the title of the current pane like this:

tmux select-pane -T "fancy pane title";
#or
tmux select-pane -t paneIndexInteger -T "fancy pane title";

If all panes have some processes running, so you can't use the command line, you can also type the commands after pressing the prefix bind (C-b by default) and a colon (:) without having "tmux" in the front of the command:

select-pane -T "fancy pane title"
#or:
select-pane -t paneIndexInteger -T "fancy pane title"

How do you implement a re-try-catch?

Spring AOP and annotation based solution:

Usage (@RetryOperation is our custom annotation for the job):

@RetryOperation(retryCount = 1, waitSeconds = 10)
boolean someMethod() throws Exception {
}

We'll need two things to accomplish this: 1. an annotation interface, and 2. a spring aspect. Here's one way to implement these:

The Annotation Interface:

import java.lang.annotation.*;

@Target(ElementType.METHOD)
@Retention(RetentionPolicy.RUNTIME)
public @interface RetryOperation {
    int retryCount();
    int waitSeconds();
}

The Spring Aspect:

import org.aspectj.lang.ProceedingJoinPoint;
import org.aspectj.lang.annotation.Around;
import org.aspectj.lang.annotation.Aspect;
import org.aspectj.lang.reflect.MethodSignature;
import org.slf4j.Logger;
import org.slf4j.LoggerFactory;
import org.springframework.stereotype.Component;
import java.lang.reflect.Method;

@Aspect @Component 
public class RetryAspect {

    private static final Logger LOGGER = LoggerFactory.getLogger(RetryAspect.class);

    @Around(value = "@annotation(RetryOperation)")
    public Object retryOperation(ProceedingJoinPoint joinPoint) throws Throwable {

        Object response = null;
        Method method = ((MethodSignature) joinPoint.getSignature()).getMethod();
        RetryOperation annotation = method.getAnnotation(RetryOperation.class);
        int retryCount = annotation.retryCount();
        int waitSeconds = annotation.waitSeconds();
        boolean successful = false;

        do {
            try {
                response = joinPoint.proceed();
                successful = true;
            } catch (Exception ex) {
                LOGGER.info("Operation failed, retries remaining: {}", retryCount);
                retryCount--;
                if (retryCount < 0) {
                    throw ex;
                }
                if (waitSeconds > 0) {
                    LOGGER.info("Waiting for {} second(s) before next retry", waitSeconds);
                    Thread.sleep(waitSeconds * 1000l);
                }
            }
        } while (!successful);

        return response;
    }
}

Get cookie by name

You can use js-cookie library to get and set JavaScript cookies.

Include to your HTML:

<script src="https://cdn.jsdelivr.net/npm/js-cookie@2/src/js.cookie.min.js"></script>

To create a Cookie:

Cookies.set('name', 'value');

To read a Cookie:

Cookies.get('name'); // => 'value'

Best way to get child nodes

Don't let white space fool you. Just test this in a console browser.

Use native javascript. Here is and example with two 'ul' sets with the same class. You don't need to have your 'ul' list all in one line to avoid white space just use your array count to jump over white space.

How to get around white space with querySelector() then childNodes[] js fiddle link: https://jsfiddle.net/aparadise/56njekdo/

var y = document.querySelector('.list');
var myNode = y.childNodes[11].style.backgroundColor='red';

<ul class="list">
    <li>8</li>
    <li>9</li>
    <li>100</li>
</ul>

<ul class="list">
    <li>ABC</li>
    <li>DEF</li>
    <li>XYZ</li>
</ul>

Format the date using Ruby on Rails

Easiest is to use strftime (docs).

If it's for use on the view side, better to wrap it in a helper, though.

Login to remote site with PHP cURL

View the source of the login page. Look for the form HTML tag. Within that tag is something that will look like action= Use that value as $url, not the URL of the form itself.

Also, while you are there, verify the input boxes are named what you have them listed as.

For example, a basic login form will look similar to:

<form method='post' action='postlogin.php'>
    Email Address: <input type='text' name='email'>
    Password: <input type='password' name='password'>
</form>

Using the above form as an example, change your value of $url to:

$url="http://www.myremotesite.com/postlogin.php";

Verify the values you have listed in $postdata:

$postdata = "email=".$username."&password=".$password;

and it should work just fine.

Generate a heatmap in MatPlotLib using a scatter data set

In Matplotlib lexicon, i think you want a hexbin plot.

If you're not familiar with this type of plot, it's just a bivariate histogram in which the xy-plane is tessellated by a regular grid of hexagons.

So from a histogram, you can just count the number of points falling in each hexagon, discretiize the plotting region as a set of windows, assign each point to one of these windows; finally, map the windows onto a color array, and you've got a hexbin diagram.

Though less commonly used than e.g., circles, or squares, that hexagons are a better choice for the geometry of the binning container is intuitive:

  • hexagons have nearest-neighbor symmetry (e.g., square bins don't, e.g., the distance from a point on a square's border to a point inside that square is not everywhere equal) and

  • hexagon is the highest n-polygon that gives regular plane tessellation (i.e., you can safely re-model your kitchen floor with hexagonal-shaped tiles because you won't have any void space between the tiles when you are finished--not true for all other higher-n, n >= 7, polygons).

(Matplotlib uses the term hexbin plot; so do (AFAIK) all of the plotting libraries for R; still i don't know if this is the generally accepted term for plots of this type, though i suspect it's likely given that hexbin is short for hexagonal binning, which is describes the essential step in preparing the data for display.)


from matplotlib import pyplot as PLT
from matplotlib import cm as CM
from matplotlib import mlab as ML
import numpy as NP

n = 1e5
x = y = NP.linspace(-5, 5, 100)
X, Y = NP.meshgrid(x, y)
Z1 = ML.bivariate_normal(X, Y, 2, 2, 0, 0)
Z2 = ML.bivariate_normal(X, Y, 4, 1, 1, 1)
ZD = Z2 - Z1
x = X.ravel()
y = Y.ravel()
z = ZD.ravel()
gridsize=30
PLT.subplot(111)

# if 'bins=None', then color of each hexagon corresponds directly to its count
# 'C' is optional--it maps values to x-y coordinates; if 'C' is None (default) then 
# the result is a pure 2D histogram 

PLT.hexbin(x, y, C=z, gridsize=gridsize, cmap=CM.jet, bins=None)
PLT.axis([x.min(), x.max(), y.min(), y.max()])

cb = PLT.colorbar()
cb.set_label('mean value')
PLT.show()   

enter image description here

SSL Error: CERT_UNTRUSTED while using npm command

Reinstall node, then update npm.

First I removed node

apt-get purge node

Then install node according to the distibution. Docs here .

Then

npm install npm@latest -g

How to check null objects in jQuery

The lookup function returns an array of matching elements. You could check if the length is zero. Note the change to only look up the elements once and reuse the results as needed.

var elem = $("#btext" + i);
if (elem.length != 0) {
   elem.text("Branch " + i);
}

Also, have you tried just using the text function -- if no element exists, it will do nothing.

$("#btext" + i).text("Branch " + i);

Dump all tables in CSV format using 'mysqldump'

mysqldump has options for CSV formatting:

--fields-terminated-by=name
                  Fields in the output file are terminated by the given
--lines-terminated-by=name
                  Lines in the output file are terminated by the given

The name should contain one of the following:

`--fields-terminated-by`

\t or "\""

`--fields-enclosed-by=name`
   Fields in the output file are enclosed by the given

and

--lines-terminated-by

  • \r
  • \n
  • \r\n

Naturally you should mysqldump each table individually.

I suggest you gather all table names in a text file. Then, iterate through all tables running mysqldump. Here is a script that will dump and gzip 10 tables at a time:

MYSQL_USER=root
MYSQL_PASS=rootpassword
MYSQL_CONN="-u${MYSQL_USER} -p${MYSQL_PASS}"
SQLSTMT="SELECT CONCAT(table_schema,'.',table_name)"
SQLSTMT="${SQLSTMT} FROM information_schema.tables WHERE table_schema NOT IN "
SQLSTMT="${SQLSTMT} ('information_schema','performance_schema','mysql')"
mysql ${MYSQL_CONN} -ANe"${SQLSTMT}" > /tmp/DBTB.txt
COMMIT_COUNT=0
COMMIT_LIMIT=10
TARGET_FOLDER=/path/to/csv/files
for DBTB in `cat /tmp/DBTB.txt`
do
    DB=`echo "${DBTB}" | sed 's/\./ /g' | awk '{print $1}'`
    TB=`echo "${DBTB}" | sed 's/\./ /g' | awk '{print $2}'`
    DUMPFILE=${DB}-${TB}.csv.gz
    mysqldump ${MYSQL_CONN} -T ${TARGET_FOLDER} --fields-terminated-by="," --fields-enclosed-by="\"" --lines-terminated-by="\r\n" ${DB} ${TB} | gzip > ${DUMPFILE}
    (( COMMIT_COUNT++ ))
    if [ ${COMMIT_COUNT} -eq ${COMMIT_LIMIT} ]
    then
        COMMIT_COUNT=0
        wait
    fi
done
if [ ${COMMIT_COUNT} -gt 0 ]
then
    wait
fi

Check for database connection, otherwise display message

Please check this:

$servername='localhost';
$username='root';
$password='';
$databasename='MyDb';

$connection = mysqli_connect($servername,$username,$password);

if (!$connection) {
die("Connection failed: " . $conn->connect_error);
}

/*mysqli_query($connection, "DROP DATABASE if exists MyDb;");

if(!mysqli_query($connection, "CREATE DATABASE MyDb;")){
echo "Error creating database: " . $connection->error;
}

mysqli_query($connection, "use MyDb;");
mysqli_query($connection, "DROP TABLE if exists employee;");

$table="CREATE TABLE employee (
id INT(6) UNSIGNED AUTO_INCREMENT PRIMARY KEY,
firstname VARCHAR(30) NOT NULL,
lastname VARCHAR(30) NOT NULL,
email VARCHAR(50),
reg_date TIMESTAMP
)"; 
$value="INSERT INTO employee (firstname,lastname,email) VALUES ('john', 'steve', '[email protected]')";
if(!mysqli_query($connection, $table)){echo "Error creating table: " . $connection->error;}
if(!mysqli_query($connection, $value)){echo "Error inserting values: " . $connection->error;}*/

How to keep two folders automatically synchronized?

You need something like this: https://github.com/axkibe/lsyncd It is a tool which combines rsync and inotify - the former is a tool that mirrors, with the correct options set, a directory to the last bit. The latter tells the kernel to notify a program of changes to a directory ot file. It says:

It aggregates and combines events for a few seconds and then spawns one (or more) process(es) to synchronize the changes.

But - according to Digital Ocean at https://www.digitalocean.com/community/tutorials/how-to-mirror-local-and-remote-directories-on-a-vps-with-lsyncd - it ought to be in the Ubuntu repository!

I have similar requirements, and this tool, which I have yet to try, seems suitable for the task.

How to check if a file exists in Go?

To check if a file doesn't exist, equivalent to Python's if not os.path.exists(filename):

if _, err := os.Stat("/path/to/whatever"); os.IsNotExist(err) {
  // path/to/whatever does not exist
}

To check if a file exists, equivalent to Python's if os.path.exists(filename):

Edited: per recent comments

if _, err := os.Stat("/path/to/whatever"); err == nil {
  // path/to/whatever exists

} else if os.IsNotExist(err) {
  // path/to/whatever does *not* exist

} else {
  // Schrodinger: file may or may not exist. See err for details.

  // Therefore, do *NOT* use !os.IsNotExist(err) to test for file existence


}

How to install ADB driver for any android device?

If no other driver package worked for your obscure device go read how to make a truly universal abd and fastboot driver out of Google's USB driver. The trick is to use CompatibleID instead of HardwareID in the driver's INF Models section

JavaScript to get rows count of a HTML table

$('tableName').find('tr').length

Plotting a 3d cube, a sphere and a vector in Matplotlib

For drawing just the arrow, there is an easier method:-

from mpl_toolkits.mplot3d import Axes3D
import matplotlib.pyplot as plt
fig = plt.figure()
ax = fig.gca(projection='3d')
ax.set_aspect("equal")

#draw the arrow
ax.quiver(0,0,0,1,1,1,length=1.0)

plt.show()

quiver can actually be used to plot multiple vectors at one go. The usage is as follows:- [ from http://matplotlib.org/mpl_toolkits/mplot3d/tutorial.html?highlight=quiver#mpl_toolkits.mplot3d.Axes3D.quiver]

quiver(X, Y, Z, U, V, W, **kwargs)

Arguments:

X, Y, Z: The x, y and z coordinates of the arrow locations

U, V, W: The x, y and z components of the arrow vectors

The arguments could be array-like or scalars.

Keyword arguments:

length: [1.0 | float] The length of each quiver, default to 1.0, the unit is the same with the axes

arrow_length_ratio: [0.3 | float] The ratio of the arrow head with respect to the quiver, default to 0.3

pivot: [ ‘tail’ | ‘middle’ | ‘tip’ ] The part of the arrow that is at the grid point; the arrow rotates about this point, hence the name pivot. Default is ‘tail’

normalize: [False | True] When True, all of the arrows will be the same length. This defaults to False, where the arrows will be different lengths depending on the values of u,v,w.

How to search in array of object in mongodb

You can do this in two ways:

  1. ElementMatch - $elemMatch (as explained in above answers)

    db.users.find({ awards: { $elemMatch: {award:'Turing Award', year:1977} } })

  2. Use $and with find

    db.getCollection('users').find({"$and":[{"awards.award":"Turing Award"},{"awards.year":1977}]})

Calculating the distance between 2 points

measure the square distance from one point to the other:

((x1-x2)*(x1-x2)+(y1-y2)*(y1-y2)) < d*d

where d is the distance, (x1,y1) are the coordinates of the 'base point' and (x2,y2) the coordinates of the point you want to check.

or if you prefer:

(Math.Pow(x1-x2,2)+Math.Pow(y1-y2,2)) < (d*d);

Noticed that the preferred one does not call Pow at all for speed reasons, and the second one, probably slower, as well does not call Math.Sqrt, always for performance reasons. Maybe such optimization are premature in your case, but they are useful if that code has to be executed a lot of times.

Of course you are talking in meters and I supposed point coordinates are expressed in meters too.

MVC 4 Data Annotations "Display" Attribute

If two different views are sharing the same model (for instance, maybe one is for mobile output and one is regular), it could be nice to have the string reside in a single place: as metadata on the ViewModel.

Additionally, if you had an inherited version of the model that necessitated a different display, it could be useful. For instance:

public class BaseViewModel
{
    [Display(Name = "Basic Name")]
    public virtual string Name { get; set; }
}

public class OtherViewModel : BaseViewModel
{
    [Display(Name = "Customized Inherited Name")]
    public override string Name { get; set; }
}

I'll admit that that example is pretty contrived...

Those are the best arguments in favor of using the attribute that I can come up with. My personal opinion is that, for the most part, that sort of thing is best left to the markup.

Xcode 6 Bug: Unknown class in Interface Builder file

This really wierd when XCode 10 stopped to "refine debugs".
In my case, i setted UITableViewDelegate and UITableViewDataSource with Storyboard and forget to implemente those required methods.
I just wanted to test the view first, before implement those methods, then crash! XCode 9, will warning about those methods, but XCode 10, really maked my code slow, not on this one, others i been thru this "error doubt".
Hope this help someone.
Thanks

jQuery change URL of form submit

Send the data from the form:

$("#change_section_type").live "change", ->
url = $(this).attr("data-url")
postData = $(this).parents("#contract_setting_form").serializeArray()
$.ajax
  type: "PUT"
  url: url
  dataType: "script"
  data: postData

Is there a C++ gdb GUI for Linux?

gdb -tui works okay if you want something GUI-ish, but still character based.

Set title background color

Try with the following code

View titleView = getWindow().findViewById(android.R.id.title);
    if (titleView != null) {
      ViewParent parent = titleView.getParent();
      if (parent != null && (parent instanceof View)) {
        View parentView = (View)parent;
        parentView.setBackgroundColor(Color.RED);
      }
    }

also use this link its very useful : http://nathanael.hevenet.com/android-dev-changing-the-title-bar-background/

How do I run a command on an already existing Docker container?

In October 2014 the Docker team introduced docker exec command: https://docs.docker.com/engine/reference/commandline/exec/

So now you can run any command in a running container just knowing its ID (or name):

docker exec -it <container_id_or_name> echo "Hello from container!"

Note that exec command works only on already running container. If the container is currently stopped, you need to first run it with the following command:

docker run -it -d shykes/pybuilder /bin/bash

The most important thing here is the -d option, which stands for detached. It means that the command you initially provided to the container (/bin/bash) will be run in the background and the container will not stop immediately.

Change directory command in Docker?

You can run a script, or a more complex parameter to the RUN. Here is an example from a Dockerfile I've downloaded to look at previously:

RUN cd /opt && unzip treeio.zip && mv treeio-master treeio && \
    rm -f treeio.zip && cd treeio && pip install -r requirements.pip

Because of the use of '&&', it will only get to the final 'pip install' command if all the previous commands have succeeded.

In fact, since every RUN creates a new commit & (currently) an AUFS layer, if you have too many commands in the Dockerfile, you will use up the limits, so merging the RUNs (when the file is stable) can be a very useful thing to do.

Get bottom and right position of an element

Instead of

var bottom = $(window).height() - link.height();
bottom = offset.top - bottom;

Why aren't you doing

var bottom = $(window).height() - top - link.height();

Edit: Your mistake is that you're doing

bottom = offset.top - bottom;

instead of

bottom = bottom - offset.top; // or bottom -= offset.top;

Setting a windows batch file variable to the day of the week

This turned out way more complex then I first suspected, and I guess that's what intrigued me, I searched every where and all the methods given wouldnt work on Windows 7.

So I have an alternate solution which uses a Visual Basic Script.

The batch creates and executes the script(DayOfWeek.vbs), assigns the scripts output (Monday, Tuesday etc) to a variable (dow), the variable is then checked and another variable (dpwnum) assigned with the days number, afterwards the VBS is deleted hope it helps:

@echo off

REM Create VBS that will get day of week in same directory as batch
echo wscript.echo WeekdayName(Weekday(Date))>>DayOfWeek.vbs

REM Cycle through output to get day of week i.e monday,tuesday etc
for /f "delims=" %%a in ('cscript /nologo DayOfWeek.vbs') do @set dow=%%a

REM delete vbs
del DayOfWeek.vbs

REM Used for testing outputs days name
echo %dow%

REM Case of the days name is important must have a capital letter at start
REM Check days name and assign value depending
IF %dow%==Monday set downum=0
IF %dow%==Tuesday set downum=1
IF %dow%==Wednesday set downum=2
IF %dow%==Thursday set downum=3
IF %dow%==Friday set downum=4
IF %dow%==Saturday set downum=5
IF %dow%==Sunday set downum=6

REM print the days number 0-mon,1-tue ... 6-sun
echo %downum%

REM set a file name using day of week number
set myfile=%downum%.bak

echo %myfile%

pause
exit

EDIT:

Though I turned to VBS, It can be done in pure batch, took me a while to get it working and a lot of searching lol, but this seems to work:

 @echo off
SETLOCAL enabledelayedexpansion
SET /a count=0
FOR /F "skip=1" %%D IN ('wmic path win32_localtime get dayofweek') DO (
    if "!count!" GTR "0" GOTO next
    set dow=%%D
    SET /a count+=1
)
:next
echo %dow%
pause

The only caveat for you on the above batch is that its day of weeks are from 1-7 and not 0-6

pycharm convert tabs to spaces automatically

For selections, you can also convert the selection using the "To spaces" function. I usually just use it via the ctrl-shift-A then find "To Spaces" from there.

How to transfer data from JSP to servlet when submitting HTML form

first up on create your jsp file : and write the text field which you want
for ex:

after that create your servlet class:

public class test{

protected void doGet(paramter , paramter){

String name  = request.getparameter("name");
 }

}

Where can I view Tomcat log files in Eclipse?

Looks like the logs are scattered? I found access logs under <ProjectLocation>\.metadata\.plugins\org.eclipse.wst.server.core\tmp0\logs

How to get back to most recent version in Git?

I am just beginning to dig deeper into git, so not sure if I understand correctly, but I think the correct answer to the OP's question is that you can run git log --all with a format specification like this: git log --all --pretty=format:'%h: %s %d'. This marks the current checked out version as (HEAD) and you can just grab the next one from the list.

BTW, add an alias like this to your .gitconfig with a slightly better format and you can run git hist --all:

  hist = log --pretty=format:\"%h %ai | %s%d [%an]\" --graph

Regarding the relative versions, I found this post, but it only talks about older versions, there is probably nothing to refer to the newer versions.

Copy / Put text on the clipboard with FireFox, Safari and Chrome

Building off the excellent answer from @David from Studio.201, this works in Safari, FF, and Chrome. It also ensures no flashing could occur from the textarea by placing it off-screen.

// ================================================================================
// ClipboardClass
// ================================================================================
var ClipboardClass = (function() {


   function copyText(text) {
    // Create temp element off-screen to hold text.
        var tempElem = $('<textarea style="position: absolute; top: -8888px; left: -8888px">');
        $("body").append(tempElem);

        tempElem.val(text).select();
        document.execCommand("copy");
        tempElem.remove();
   }


    // ============================================================================
   // Class API
   // ============================================================================
    return {
        copyText: copyText
    };
})();

C++ error 'Undefined reference to Class::Function()'

Specify the Class Card for the constructor-:

void Card::Card(Card::Rank rank, Card::Suit suit) {

And also define the default constructor and destructor.

Setting ANDROID_HOME enviromental variable on Mac OS X

To set ANDROID_HOME, variable, you need to know how you installed android dev setup.

If you don't know you can check if the following paths exist in your machine. Add the following to .bashrc, .zshrc, or .profile depending on what you use

If you installed with homebrew,

export ANDROID_HOME=/usr/local/opt/android-sdk

Check if this path exists:

If you installed android studio following the website,

export ANDROID_HOME=~/Library/Android/sdk

Finally add it to path:

export PATH=$PATH:$ANDROID_HOME/tools:$ANDROID_HOME/platform-tools

If you're too lazy to open an editor do this:

echo "export ANDROID_HOME=~/Library/Android/sdk" >> ~/.bashrc
echo "export PATH=$PATH:$ANDROID_HOME/tools:$ANDROID_HOME/platform-tools" >> ~/.bashrc

How to print the current time in a Batch-File?

This works with Windows 10, 8.x, 7, and possibly further back:

@echo Started: %date% %time%
.
.
.
@echo Completed: %date% %time%

Using request.setAttribute in a JSP page

Correct me if wrong...I think request does persist between consecutive pages..

Think you traverse from page 1--> page 2-->page 3.

You have some value set in the request object using setAttribute from page 1, which you retrieve in page 2 using getAttribute,then if you try setting something again in same request object to retrieve it in page 3 then it fails giving you null value as "the request that created the JSP, and the request that gets generated when the JSP is submitted are completely different requests and any attributes placed on the first one will not be available on the second".

I mean something like this in page 2 fails:

Where as the same thing has worked in case of page 1 like:

So I think I would need to proceed with either of the two options suggested by Phill.

What do raw.githubusercontent.com URLs represent?

There are two ways of looking at github content, the "raw" way and the "Web page" way.

raw.githubusercontent.com returns the raw content of files stored in github, so they can be downloaded simply to your computer. For example, if the page represents a ruby install script, then you will get a ruby install script that your ruby installation will understand.

If you instead download the file using the github.com link, you will actually be downloading a web page with buttons and comments and which displays your wanted script in the middle -- it's what you want to give to your web browser to get a nice page to look at, but for the computer, it is not a script that can be executed or code that can be compiled, but a web page to be displayed. That web page has a button called Raw that sends you to the corresponding content on raw.githubusercontent.com.

To see the content of raw.githubusercontent.com/${repo}/${branch}/${path} in the usual github interface:

  1. you replace raw.githubusercontent.com with plain github.com
  2. AND you insert "blob" between the repo name and the branch name.

In this case, the branch name is "master" (which is a very common branch name), so you replace /master/ with /blob/master/, and so

https://raw.githubusercontent.com/Homebrew/install/master/install

becomes

https://github.com/Homebrew/install/blob/master/install

This is the reverse of finding a file on Github and clicking the Raw link.

How to capitalize the first letter of a String in Java?

thanks I have read some of the comments and I came with the following

public static void main(String args[]) 
{
String myName = "nasser";
String newName = myName.toUpperCase().charAt(0) +  myName.substring(1);
System.out.println(newName );
}

I hope its helps good luck

Git pull command from different user

Your question is a little unclear, but if what you're doing is trying to get your friend's latest changes, then typically what your friend needs to do is to push those changes up to a remote repo (like one hosted on GitHub), and then you fetch or pull those changes from the remote:

  1. Your friend pushes his changes to GitHub:

    git push origin <branch>
    
  2. Clone the remote repository if you haven't already:

    git clone https://[email protected]/abc/theproject.git
    
  3. Fetch or pull your friend's changes (unnecessary if you just cloned in step #2 above):

    git fetch origin
    git merge origin/<branch>
    

    Note that git pull is the same as doing the two steps above:

    git pull origin <branch>
    

See Also

How to add items into a numpy array

One way to do it (may not be the best) is to create another array with the new elements and do column_stack. i.e.

>>>a = array([[1,3,4],[1,2,3]...[1,2,1]])
[[1 3 4]
 [1 2 3]
 [1 2 1]]

>>>b = array([1,2,3])
>>>column_stack((a,b))
array([[1, 3, 4, 1],
       [1, 2, 3, 2],
       [1, 2, 1, 3]])

Excel Create Collapsible Indented Row Hierarchies

A much easier way is to go to Data and select Group or Subtotal. Instant collapsible rows without messing with pivot tables or VBA.

How to control the width and height of the default Alert Dialog in Android?

longButton.setOnClickListener {
  show(
    "1\n2\n3\n4\n5\n6\n7\n8\n9\n0\n" +
      "1\n2\n3\n4\n5\n6\n7\n8\n9\n0\n" +
      "1\n2\n3\n4\n5\n6\n7\n8\n9\n0\n" +
      "1\n2\n3\n4\n5\n6\n7\n8\n9\n0\n" +
      "1\n2\n3\n4\n5\n6\n7\n8\n9\n0\n" +
      "1\n2\n3\n4\n5\n6\n7\n8\n9\n0\n" +
      "1234567890-12345678901234567890123456789012345678901234567890"
  )
}

shortButton.setOnClickListener {
  show(
    "1234567890\n" +
      "1234567890-12345678901234567890123456789012345678901234567890"
  )
}

private fun show(msg: String) {
  val builder = AlertDialog.Builder(this).apply {
    setPositiveButton(android.R.string.ok, null)
    setNegativeButton(android.R.string.cancel, null)
  }

  val dialog = builder.create().apply {
    setMessage(msg)
  }
  dialog.show()

  dialog.window?.decorView?.addOnLayoutChangeListener { v, _, _, _, _, _, _, _, _ ->
    val displayRectangle = Rect()
    val window = dialog.window
    v.getWindowVisibleDisplayFrame(displayRectangle)
    val maxHeight = displayRectangle.height() * 0.6f // 60%

    if (v.height > maxHeight) {
      window?.setLayout(window.attributes.width, maxHeight.toInt())
    }
  }
}

short message

long message

React-router urls don't work when refreshing or writing manually

Looking at the comments on the accepted answer and the generic nature of this question ('don't work'), I thought this might be a good place for some general explanations about the issues involved here. So this answer is intended as background info / elaboration on the specific use case of the OP. Please bear with me.

Server-side vs Client-side

The first big thing to understand about this is that there are now 2 places where the URL is interpreted, whereas there used to be only 1 in 'the old days'. In the past, when life was simple, some user sent a request for http://example.com/about to the server, which inspected the path part of the URL, determined the user was requesting the about page, and then sent back that page.

With client-side routing, which is what React-Router provides, things are less simple. At first, the client does not have any JS code loaded yet. So the very first request will always be to the server. That will then return a page that contains the needed script tags to load React and React Router etc. Only when those scripts have loaded does phase 2 start. In phase 2, when the user clicks on the 'About us' navigation link, for example, the URL is changed locally only to http://example.com/about (made possible by the History API), but no request to the server is made. Instead, React Router does its thing on the client-side, determines which React view to render, and renders it. Assuming your about page does not need to make any REST calls, it's done already. You have transitioned from Home to About Us without any server request having fired.

So basically when you click a link, some Javascript runs that manipulates the URL in the address bar, without causing a page refresh, which in turn causes React Router to perform a page transition on the client-side.

But now consider what happens if you copy-paste the URL in the address bar and e-mail it to a friend. Your friend has not loaded your website yet. In other words, she is still in phase 1. No React Router is running on her machine yet. So her browser will make a server request to http://example.com/about.

And this is where your trouble starts. Until now, you could get away with just placing a static HTML at the webroot of your server. But that would give 404 errors for all other URLs when requested from the server. Those same URLs work fine on the client-side, because there React Router is doing the routing for you, but they fail on the server-side unless you make your server understand them.

Combining server- and client-side routing

If you want the http://example.com/about URL to work on both the server- and the client-side, you need to set up routes for it on both the server- and the client-side. Makes sense right?

And this is where your choices begin. Solutions range from bypassing the problem altogether, via a catch-all route that returns the bootstrap HTML, to the full-on isomorphic approach where both the server and the client run the same JS code.

.

Bypassing the problem altogether: Hash History

With Hash History instead of Browser History, your URL for the about page would look something like this: http://example.com/#/about The part after the hash (#) symbol is not sent to the server. So the server only sees http://example.com/ and sends the index page as expected. React-Router will pick up the #/about part and show the correct page.

Downsides:

  • 'ugly' URLs
  • Server-side rendering is not possible with this approach. As far as Search Engine Optimization (SEO) is concerned, your website consists of a single page with hardly any content on it.

.

Catch-all

With this approach, you do use Browser History but just set up a catch-all on the server that sends /* to index.html, effectively giving you much the same situation as with Hash History. You do have clean URLs however and you could improve upon this scheme later without having to invalidate all your user's favorites.

Downsides:

  • More complex to set up
  • Still no good SEO

.

Hybrid

In the hybrid approach, you expand upon the catch-all scenario by adding specific scripts for specific routes. You could make some simple PHP scripts to return the most important pages of your site with content included, so Googlebot can at least see what's on your page.

Downsides:

  • Even more complex to set up
  • Only good SEO for those routes you give the special treatment
  • Duplicating code for rendering content on server and client

.

Isomorphic

What if we use Node JS as our server so we can run the same JS code on both ends? Now, we have all our routes defined in a single react-router config and we don't need to duplicate our rendering code. This is 'the holy grail' so to speak. The server sends the exact same markup as we would end up with if the page transition had happened on the client. This solution is optimal in terms of SEO.

Downsides:

  • Server must (be able to) run JS. I've experimented with Java i.c.w. Nashorn but it's not working for me. In practice, it mostly means you must use a Node JS based server.
  • Many tricky environmental issues (using window on server-side etc)
  • Steep learning curve

.

Which should I use?

Choose the one that you can get away with. Personally, I think the catch-all is simple enough to set up, so that would be my minimum. This setup allows you to improve on things over time. If you are already using Node JS as your server platform, I'd definitely investigate doing an isomorphic app. Yes, it's tough at first, but once you get the hang of it it's actually a very elegant solution to the problem.

So basically, for me, that would be the deciding factor. If my server runs on Node JS, I'd go isomorphic; otherwise, I would go for the Catch-all solution and just expand on it (Hybrid solution) as time progresses and SEO requirements demand it.

If you'd like to learn more about isomorphic (also called 'universal') rendering with React, there are some good tutorials on the subject:

Also, to get you started, I recommend looking at some starter kits. Pick one that matches your choices for the technology stack (remember, React is just the V in MVC, you need more stuff to build a full app). Start with looking at the one published by Facebook itself:

Or pick one of the many by the community. There is a nice site now that tries to index all of them:

I started with these:

Currently, I am using a home-brew version of universal rendering that was inspired by the two starter kits above, but they are out of date now.

Good luck with your quest!

How to write multiple conditions of if-statement in Robot Framework

The below code worked fine:

Run Keyword if    '${value1}' \ \ == \ \ '${cost1}' \ and \ \ '${value2}' \ \ == \ \ 'cost2'    LOG    HELLO

Multiple SQL joins

 SELECT
 B.Title, B.Edition, B.Year, B.Pages, B.Rating     --from Books
, C.Category                                        --from Categories
, P.Publisher                                       --from Publishers
, W.LastName                                        --from Writers

FROM Books B

JOIN Categories_Books CB ON B._ISBN = CB._Books_ISBN
JOIN Categories_Books CB ON CB.__Categories_Category_ID = C._CategoryID
JOIN Publishers P ON B.PublisherID = P._Publisherid
JOIN Writers_Books WB ON B._ISBN = WB._Books_ISBN
JOIN Writers W ON WB._Writers_WriterID = W._WriterID

How to list running screen sessions?

The command screen -list may be what you want.

See the man

How do I get sed to read from standard input?

use "-e" to specify the sed-expression

cat input.txt | sed -e 's/foo/bar/g'

PHP - Copy image to my server direct from URL

Two ways, if you're using PHP5 (or higher)

copy('http://www.google.co.in/intl/en_com/images/srpr/logo1w.png', '/tmp/file.png');

If not, use file_get_contents

//Get the file
$content = file_get_contents("http://www.google.co.in/intl/en_com/images/srpr/logo1w.png");
//Store in the filesystem.
$fp = fopen("/location/to/save/image.png", "w");
fwrite($fp, $content);
fclose($fp);

From this SO post

Temporary table in SQL server causing ' There is already an object named' error

You are dropping it, then creating it, then trying to create it again by using SELECT INTO. Change to:

DROP TABLE #TMPGUARDIAN
CREATE TABLE #TMPGUARDIAN(
LAST_NAME NVARCHAR(30),
FRST_NAME NVARCHAR(30))  

INSERT INTO #TMPGUARDIAN 
SELECT LAST_NAME,FRST_NAME  
FROM TBL_PEOPLE

In MS SQL Server you can create a table without a CREATE TABLE statement by using SELECT INTO

Cannot declare instance members in a static class in C#

As John Weldon said all members must be static in a static class. Try

public static class employee
{
     static NameValueCollection appSetting = ConfigurationManager.AppSettings;    

}

Javascript callback when IFRAME is finished loading?

I am using jQuery and surprisingly this seems to load as I just tested and loaded a heavy page and I didn't get the alert for a few seconds until I saw the iframe load:

$('#the_iframe').load(function(){
    alert('loaded!');
});

So if you don't want to use jQuery take a look at their source code and see if this function behaves differently with iframe DOM elements, I will look at it myself later as I am interested and post here. Also I only tested in the latest chrome.

How do I access ViewBag from JS

<script type="text/javascript">
      $(document).ready(function() {
                showWarning('@ViewBag.Message');
      });

</script>

You can use ViewBag.PropertyName in javascript like this.

How to handle back button in activity

You should use:

@Override
public boolean onKeyDown(int keyCode, KeyEvent event)  {
    if (android.os.Build.VERSION.SDK_INT < android.os.Build.VERSION_CODES.ECLAIR
            && keyCode == KeyEvent.KEYCODE_BACK
            && event.getRepeatCount() == 0) {
        // Take care of calling this method on earlier versions of
        // the platform where it doesn't exist.
        onBackPressed();
    }

    return super.onKeyDown(keyCode, event);
}

@Override
public void onBackPressed() {
    // This will be called either automatically for you on 2.0
    // or later, or by the code above on earlier versions of the
    // platform.
    return;
}

As defined here: http://android-developers.blogspot.com/2009/12/back-and-other-hard-keys-three-stories.html

If you are using an older version to compile the code, replace android.os.Build.VERSION_CODES.ECLAIR by 5 (you can add a private int named ECLAIR for example)

Remove the complete styling of an HTML button/submit

Try removing the border from your button:

input, button, submit
{
  border:none;
}

How can I alias a default import in JavaScript?

defaultMember already is an alias - it doesn't need to be the name of the exported function/thing. Just do

import alias from 'my-module';

Alternatively you can do

import {default as alias} from 'my-module';

but that's rather esoteric.

Div not expanding even with content inside

You didn't typed the closingtag from the div with id="infohold.

Can anyone confirm that phpMyAdmin AllowNoPassword works with MySQL databases?

After lots of struggle I found here you go:

  1. open folder -> xampp
  2. then open -> phpmyadmin
  3. finally open -> config.inc
$cfg['blowfish_secret'] = ''; /* YOU MUST FILL IN THIS FOR COOKIE AUTH! */
  1. Put SHA256 inside single quotations like this one

830bbca930d5e417ae4249931838e2c70ca0365044268fa0ede75e33aff677de

$cfg['blowfish_secret'] = '830bbca930d5e417ae4249931838e2c70ca0365044268fa0ede75e33aff677de
';

I found this when I was downloading updated version of phpmyadmin. I wish this solution help you. enter image description here

Is there a way to use shell_exec without waiting for the command to complete?

Sure, for windows you can use:

$WshShell = new COM("WScript.Shell");
$oExec = $WshShell->Run("C:/path/to/php-win.exe -f C:/path/to/script.php", 0, false);

Note:

If you get a COM error, add the extension to your php.ini and restart apache:

[COM_DOT_NET]
extension=php_com_dotnet.dll

How to implement a material design circular progress bar in android

The platform uses a vector drawable, so you can't reuse it as in in older versions.
However, the support lib v4 contains a backport of this drawable : http://androidxref.com/5.1.0_r1/xref/frameworks/support/v4/java/android/support/v4/widget/MaterialProgressDrawable.java It has a @hide annotation (it is here for the SwipeRefreshLayout), but nothing prevents you from copying this class in your codebase.

mean() warning: argument is not numeric or logical: returning NA

From R 3.0.0 onwards mean(<data.frame>) is defunct (and passing a data.frame to mean will give the error you state)

A data frame is a list of variables of the same number of rows with unique row names, given class "data.frame".

In your case, result has two variables (if your description is correct) . You could obtain the column means by using any of the following

lapply(results, mean, na.rm = TRUE)
sapply(results, mean, na.rm = TRUE)
colMeans(results, na.rm = TRUE)

How to have a a razor action link open in a new tab?

If your goal is to use the ActionLink helper and open a new tab:

@Html.ActionLink("New tab please", "Home", null , new { target = "_blank" })

@Html.ActionLink("New tab please", "Home", Nothing, New With {Key .target = "_blank"})

How to make a .jar out from an Android Studio project

.jar file will be automatically generate when u compile/run your application.

You can find your class.jar file from root_folder/app/build/intermediates/bundles/debug

jar file location

jquery live hover

As of jQuery 1.4.1, the hover event works with live(). It basically just binds to the mouseenter and mouseleave events, which you can do with versions prior to 1.4.1 just as well:

$("table tr")
    .mouseenter(function() {
        // Hover starts
    })
    .mouseleave(function() {
        // Hover ends
    });

This requires two binds but works just as well.

How to turn NaN from parseInt into 0 for an empty string?

an helper function which still allow to use the radix

function parseIntWithFallback(s, fallback, radix) {
    var parsed = parseInt(s, radix);
    return isNaN(parsed) ? fallback : parsed;
}

How to sort a List of objects by their date (java collections, List<Object>)

I'd add Commons NullComparator instead to avoid some problems...

Composer killed while updating

You can try setting preferred-install to "dist" in Composer config.

remove space between paragraph and unordered list

This simple way worked fine for me:

<ul style="margin-top:-30px;">

How to save S3 object to a file using boto3

boto3 now has a nicer interface than the client:

resource = boto3.resource('s3')
my_bucket = resource.Bucket('MyBucket')
my_bucket.download_file(key, local_filename)

This by itself isn't tremendously better than the client in the accepted answer (although the docs say that it does a better job retrying uploads and downloads on failure) but considering that resources are generally more ergonomic (for example, the s3 bucket and object resources are nicer than the client methods) this does allow you to stay at the resource layer without having to drop down.

Resources generally can be created in the same way as clients, and they take all or most of the same arguments and just forward them to their internal clients.

How to parse a JSON string to an array using Jackson

I finally got it:

ObjectMapper objectMapper = new ObjectMapper();
TypeFactory typeFactory = objectMapper.getTypeFactory();
List<SomeClass> someClassList = objectMapper.readValue(jsonString, typeFactory.constructCollectionType(List.class, SomeClass.class));

How Big can a Python List Get?

It varies for different systems (depends on RAM). The easiest way to find out is

import six six.MAXSIZE 9223372036854775807 This gives the max size of list and dict too ,as per the documentation

ASP.NET MVC Page Won't Load and says "The resource cannot be found"

Open your Controller.cs file and near your public ActionResult Index(), in place of Index write the name of your page you want to run in the browser. For me it was public ActionResult Login().

A simple jQuery form validation script

you can use jquery validator for that but you need to add jquery.validate.js and jquery.form.js file for that. after including validator file define your validation something like this.

<script type="text/javascript">
$(document).ready(function(){
    $("#formID").validate({
    rules :{
        "data[User][name]" : {
            required : true
        }
    },
    messages :{
        "data[User][name]" : {
            required : 'Enter username'
        }
    }
    });
});
</script>

You can see required : true same there is many more property like for email you can define email : true for number number : true

Generator expressions vs. list comprehensions

I'm using the Hadoop Mincemeat module. I think this is a great example to take a note of:

import mincemeat

def mapfn(k,v):
    for w in v:
        yield 'sum',w
        #yield 'count',1


def reducefn(k,v): 
    r1=sum(v)
    r2=len(v)
    print r2
    m=r1/r2
    std=0
    for i in range(r2):
       std+=pow(abs(v[i]-m),2)  
    res=pow((std/r2),0.5)
    return r1,r2,res

Here the generator gets numbers out of a text file (as big as 15GB) and applies simple math on those numbers using Hadoop's map-reduce. If I had not used the yield function, but instead a list comprehension, it would have taken a much longer time calculating the sums and average (not to mention the space complexity).

Hadoop is a great example for using all the advantages of Generators.

Cannot find mysql.sock

My problem was also the mysql.sock-file.

During the drupal installation process, i had to say which database i want to use but my database wasn't found

mkdir /var/mysql
ln -s /tmp/mysql.sock /var/mysql/mysql.sock

the system is searching mysql.sock but it's in the wrong directory

all you have to do is to link it ;)

it took me a lot of time to google all important informations but it took me even hours to find out how to adapt , but now i can present the result :D

ps: if you want to be exactly you have to link your /tmp/mysql.sock-file (if it is located in your system there too) to the directory given by the php.ini (or php.default.ini) where pdo_mysql.default_socket= ...

Kotlin: How to get and set a text to TextView in Android using Kotlin?

import kotlinx.android.synthetic.main.MainActivity.*

class Mainactivity : AppCompatActivity() {


    override fun onCreate(savedInstanceState: Bundle?) {
        super.onCreate(savedInstanceState)
        setContentView(R.layout.MainActivity)

        txt.setText("hello Kotlin")

    }

}

How to change TextBox's Background color?

If it's WPF, there is a collection of colors in the static class Brushes.

TextBox.Background = Brushes.Red;

Of course, you can create your own brush if you want.

LinearGradientBrush myBrush = new LinearGradientBrush();
myBrush.GradientStops.Add(new GradientStop(Colors.Yellow, 0.0));
myBrush.GradientStops.Add(new GradientStop(Colors.Orange, 0.5));
myBrush.GradientStops.Add(new GradientStop(Colors.Red, 1.0));
TextBox.Background = myBrush;